INFECTOLOGÍA Flashcards

1
Q

Mycobacterium tuberculosis ( como bacteria)
-menciona sus caracteristicas microbiologicas principales
-menciona sus mecanismo principal de accion al entrar al huesped
-menciona brevemente su epidemiologia mundial

A

bacilo aerobio inmovil no espurulados, se tiñe debil como G+, es acidorresistente
pared rica en lipidos, de crecimiento lento multiresistente
sus proteinas y porinas activan respuesta inmune
patogeno intraceular, transmintido por particulas respiratorias
inhibe union de fagolisosoma con lisosoma, atrae a macrofagos circulantes por C5a y se generan celulas gigantes multinucleadas “langhans)
macrofagos infectados se pueden diseminar a gangliosm, medula, bazo, riñon SNC, higado
se activan principalmente en huespedes sobreinfectados o que en algun momento se inmunodeprimen (VIH, tx inmunosupresores, adictos etc)
principalmente en sureste asiatico, africa subsahariana y este de europa
1/3 de pa poblacion mundial infectada

How well did you know this?
1
Not at all
2
3
4
5
Perfectly
2
Q

TUBERCULOSIS PRIMARIA
-Menciona caracteristicas basicas de la clinica, el diagnostico inicial y los principales hallazgos radiograficos

A

-Mayoria de casos son asintomaticos
-dx por prueba cutanea en su mayoria
-clincia: acortamiento de respiracion, tos no productiva, eritema nodoso, crepitaciones y sibilancias focales
-rx con opacidad, linfadenopatia hiliar,colapso lobulo superior, derrame transitorio
-Ghon: latencia clinica
-Ranke: calcificaciones ganglionares hiliares-paratraqueales

How well did you know this?
1
Not at all
2
3
4
5
Perfectly
3
Q

Reactivacion de la tuberculosis (tb posprimaria)
-menciona las caracteristicas clinicas, radiologicas basicas y presentacion mas comun

A

-Inicia por hipersensibilodad retardada
-TB pulmonar es la mas comun
- Tos productiva (esputo hematico), diaforesis nocturna, anorexia, prdida pondera, fiebre
– dolor pleuritico (puede haber, no es regla)
- matidez a la percusion toracica, ruidos respiratorios anforicos, crepitaciones
-Rx: lesiones en segmentos apicales, de lobulo superior, mprogresion de opacidad parcheada, consolidacion y cavitacion

How well did you know this?
1
Not at all
2
3
4
5
Perfectly
4
Q

Reinfeccion de tuberculosis

A

Es indistinguible clínicamente
se puede diagnosticar por fracaso terapéutico previo (cambio de suseptibilidad en la terapeutica) o examen de DNA

How well did you know this?
1
Not at all
2
3
4
5
Perfectly
5
Q

Tuberculosis extrapulmonar

A

-Aproximadamente 20% de pacientes con VIH
-En niños y adultos jovenes areas endemicas suele ser enfermedad primaria progresiva
-en ancianos prevalencia baja suele ser manifestacion de reactivacion

How well did you know this?
1
Not at all
2
3
4
5
Perfectly
6
Q

Tuberculosis pleural
-Menciona su principal diseminacion, sintomas principales y caracteristicas clinicas

A

-Inicia por foco subpleural o por diseminacion hematogena
-Inicio abrupto de fiebre, dolor pleuritico, tos
-matidez y disminucion de ruids respiratorios
-derrame pleural (generalmente unilateral de lado derecho)

How well did you know this?
1
Not at all
2
3
4
5
Perfectly
7
Q

Tuberculosis miliar
Menciona caractristiacas clinicas principales

A

-Inicio insidioso
-Fiebre, perdida ponderal, diaforesis nocturna
POCAS MANIFESTACIONES DE LOCALIZACION
-tubérculos coroideos, linfadenopatia y hepatomegalia
-TAC se prefiere por ser mas sensible
+TB diseminada arreactiva suele encontrarse en VIH, Inmuno avanzada o hemocultivo + puede ser unica manifestacion

How well did you know this?
1
Not at all
2
3
4
5
Perfectly
8
Q

Meningitis tuberculosa

A

-Fiebre, cefalea, meningismo con -2semanas de evolucion
-Depresion nivel de alerta, diplopia, hemiparesia (rara vez)
-EF: rigidez nucal, neuropatia craneal de VI, II, IV, VII (ese es el orden de frecuencia al afectarse)
-RX: toracica compatible con tb miliar o pulmonar
-TAC craneo: aumento contraste en meninges basilares, areas hipodensas correspondiente a infarto, hidrocefalea, tuberculomas (lesion inflamatoria focal)

How well did you know this?
1
Not at all
2
3
4
5
Perfectly
9
Q

linfadenitis tuberculosa: menciona los pacientes principales que se encuentran afectados y zona afectada

A

Se acompaña generalmente de tb pulmonar en pacientes con vih
ganglios supraclaviculares y cervicales posteriores
DOLOROSO
Biopsia por aspiracion es el mejor aboordaje diagnostico

How well did you know this?
1
Not at all
2
3
4
5
Perfectly
10
Q

PERICARDITIS TUBERCULOSA
menciona la clinica principal y el manejo de eleccion

A

-Evolucion cronica, fiebre, diaforiesis nocturna, dolor toracico, acortamiento de la respiracion, edemapedio, ICDERECHA
-Biopsia y aspiracion pericardica son procesos dx de eleccion
en taponamiento (10%) de los casos es dx y terapeutica

How well did you know this?
1
Not at all
2
3
4
5
Perfectly
11
Q

PERITONITIS TUBERCULOSA
menciona las caracteristicas y el metodo diagnostico

A

-Dolor abdominal, fiebre, edema, diaforesis noctirna, perdida ponderal, ascitis trasudado
-biopsia peritoneal laparoscopica es el metodo dx por eleccion cuando hay ascitis

How well did you know this?
1
Not at all
2
3
4
5
Perfectly
12
Q

TUBERCULOSIS GASTROINTESTINAL
Menciona sus caracteristicas clinicas y abordaje dx

A

-Fiebre, dolor abdominal, sangrado/obstruccion,
-dx con sospecha clinica, tac y en algunos casos biopsia de ileon.

How well did you know this?
1
Not at all
2
3
4
5
Perfectly
13
Q

TUBERCULOSIS RENAL
-menciona clinica, hallazgos de imagen para el diagnostico

A

-Se manifiesta como disuria, hematuria, y dolor en flancos
-Dx empieza en hallazgo de piuira esteril o hematuria
-Pielografia o TAC revelan lesiones corticales, masas o lesiones cavitarias, lesion-necrosis de los calices o los ureteres

How well did you know this?
1
Not at all
2
3
4
5
Perfectly
14
Q

OSTEOMIELITIS VERTEBRAL
-Menciona sitios de presentacion,y caracteristicas clincias

A

Foco inicial: region subcondral
-Se presennta principalmente en vertebras toracicas bajas o lumbares
-dolor radicular, dolor de espalda, debilidad en extremidades inferiores, incontincencia (casos severos)

How well did you know this?
1
Not at all
2
3
4
5
Perfectly
15
Q

TUBERCULOSIS GENITAL FEMENINA
Minciona sitios mas frecuenntes y su clinica basica

A
  • foco: endosalpinx
    disemina a: endometrio 50%, ovarios 30 %, cervix 10%, vagina 1%.
    lesion en cervix puede asemejar a carcinoma
    -Los sintomas sistemicos son infrecuentes
    puede presentarse como EPI no remitente a tratamiento
How well did you know this?
1
Not at all
2
3
4
5
Perfectly
16
Q

TUBERCULOSIS
Menciona en que semanas se puede hacerPPD y cuanto tiempo debe de ser interpretable

A

PPD se produce de 3-4 semanas posterior a la infeccion y debe de interpretarse 72 horas despues
en pacientes con VIH sin antirv, debe repetirse la prieba cuando el conteo alcance 200/ml de cd4 y anualmente a partir de ahi

How well did you know this?
1
Not at all
2
3
4
5
Perfectly
17
Q

TUBERCULOSIS
¿cuales son los criterios para definir positividad a PPD?

A

— += 5mm en: VIH, pacientes con prednisona >15 mg dia, transplantados, contacto estrecho de pacientes con tb, placas rx anomalas previas compatibles con tb, <5 años, desnutridos
— += 10 mm resto

How well did you know this?
1
Not at all
2
3
4
5
Perfectly
18
Q

TUBERCULOSIS
menciona las tinciones clasicas y el estandar de oro para dx

A

ziehl neelsen, kinyoun
-cultivo con huevo o agar (lowenstein-jensen; middlebrook)
-caldos de cultivo especiales 10-14 dias de incubacion

How well did you know this?
1
Not at all
2
3
4
5
Perfectly
19
Q

TUBERCULOSIS
en mexico, cuales son los comorbidos mas asociados a enfermedad por tb

A
  • Diabetes 20%
    -Desnutricion 13%
    -VIH 10%
    -Alcoholismo 3%
How well did you know this?
1
Not at all
2
3
4
5
Perfectly
20
Q

TUBERCULOSIS
exolica la relacion simbiotica de vih con la tb

A

La TB es la enfermedad infecciosa por la que fallece el mayor numero de personas que viven con VIH y el VIH es quien activa la TB latente de una persona infectada

How well did you know this?
1
Not at all
2
3
4
5
Perfectly
21
Q

TUBERCULOSIS
cual es la terapia preventiva para tb latente

A

isoniacida 5-10mgxkg dia peso sin superar 300mg dia (una toma)
-por 6 meses en contactos <5 años con o sin vacuna cuando se descarto enfermedad tb
-por 6 meses en contactos 5-14 años no vacunados con tb descartada
- por 6 meses en contacos menores o mayores de 15 años en inmunocomprometidos

How well did you know this?
1
Not at all
2
3
4
5
Perfectly
22
Q

TUBERCULOSIS
como debe de realizarse la baciloscopia por expectoracion

A

sintomaticos respiratorios o casos probables
-Tres muestras de espectoracion seriadas
-Si no se confirma pero hay clinica o signos rx, se deben repetir las tres seriadas
-en tb confirmada debe repetrise mensualmente durante el tratamiento

How well did you know this?
1
Not at all
2
3
4
5
Perfectly
23
Q

TUBERCULOSIS
cuando se deben de solicitar los cultuivos

A

-Dx o sospecha clinica o radiologica con resultado negativo de 3 baciloscopias
-sospecha de TB extrapulmonar
-vih
-niños
-baciloscopia persistente al segundo mes
-tb previamente tratada
-contacto con TBresistente
-recaida/fracaso de tto

How well did you know this?
1
Not at all
2
3
4
5
Perfectly
24
Q

TUBERCULOSIS
¿Cuales son los farmacos de primera linea, su mecanismo de accion y pricipales efectos adversos?

A
  • isoniacida: bactericida intra y extra cel, neuritis, hepatitis, sxlupoide
  • Rifampicina: bactericida, hepatitis, reaccion febril, purpura,
  • Pirazinamida: bactericida intracel esterilizante, hiperuricemia, hepatitis, vomitos, hipersensibilidad cutenea
    -Etambutol: bactericida intra y extraccel, neuritis optica
    -Estreptomicina: bactericida extraccel, bloqueo neuromiscular, lesion del VII par, nefrotoxico
How well did you know this?
1
Not at all
2
3
4
5
Perfectly
25
Q

TUBERCULOSIS
menciona el tratamiento de primera linea primario acortado

A

intensiva: Lunes a sabado hasta completar 60 dosis, 1 toma
R: 600MG
H: 300MG
Z: 1,500-2000MG
E: 1,200MG
Sosten : intermitente L-M-V hasta completar 45 dosis. 1 toma
H: 800MG
R: 600MG

How well did you know this?
1
Not at all
2
3
4
5
Perfectly
26
Q

TUBERCULOSIS
Menciona cual es el retratamiento primario

A

INTENSIVA: diario L-S 60 dosis. 1 toma
R: 600MG
H: 300MG
Z: 1,500-2000MG
E: 1,200MG
S: 1,000MG
INTERMEDIA: diario L-S 30 dosis. 1 toma
R: 600MG
H: 300MG
Z: 1,500-2000MG
E: 1,200MG
SOSTEN L-M-V hasta completar 60 dosis
R: 800MG
H: 600MG
E: 1,200MG

How well did you know this?
1
Not at all
2
3
4
5
Perfectly
27
Q

TUBERCULOSIS
¿Durante cuanto tiempo debe de darse seguimiento a pacientes curados?

A

Semestralmente al menos durante dos años

How well did you know this?
1
Not at all
2
3
4
5
Perfectly
28
Q

TUBERCULOSIS
menciona los antimicrobianos de segunda linea

A

kanamicina KM
AMIKACINA AM
CAPREOMICINA CM
ETIONAMIDA ETO
PROTIONAMIDA PTO
OFLOXACINA OFX
LEVOFLOXACINA LFX
MOXIFLOXACINA MFX
CICLOSERINA CS

How well did you know this?
1
Not at all
2
3
4
5
Perfectly
29
Q

TUBERCULOSIS
En pacientes que se diagnotican tb y vih al mismo tiempo, cuanto debe iniciarse la terapia antiretroviral

A

se debe iniciar entre 2-8 semanas posteriores al inicio de tratamiento con antituberculosos para evitar efctos adversos, interacciones farmacologicas o sindrome inflamatorio de reconstitucion inmunologica

How well did you know this?
1
Not at all
2
3
4
5
Perfectly
30
Q

STAPHYLOCOCCUS AUREUS
Menciona los componentes de su estructura

A

Coco gram+ en forma de racimo de uvas. Se encuentra piel y mucosas, especialmente narina, nasofaringe y pliegues cutáneos húmedos. Catalasa y coagulasa +
- Cápsula: inhibe fagocitosis y quimiotaxis; facilita la adherencia a tejidos.
- Peptidoglucano: endotoxina + estabilidad osmotica
- Ácido teicoico: regula la concentración de cationes y media la adhesión por su unión a fibronectina.
- Proteína A: previene la respuesta mediada por anticuerpos.
- Toxinas: citotoxinas (⍺, β, γ, δ); exofialtivas (ETA, ETB); enterotoxinas (A y B); toxina-1 del sx de choque tóxico/ exotoxina pirogénica.
Enzimas: coaguladas, catalasa y peniciclinasa.

How well did you know this?
1
Not at all
2
3
4
5
Perfectly
31
Q

MICOBACTERIUM AVIUM
menciona caracteristicas microbiologicas y epidemiologicas basicas

A

-Bacilos gram+ acidoresistente de crecimiento intracelular
-Distribucion universal, la enfermedad se manifiesta mas en paises donde TB es menos frecuente
-Agua y alimentos contaminados
-Inmunodeprimidos o en pacients con enfermedades pulmonares largas

How well did you know this?
1
Not at all
2
3
4
5
Perfectly
32
Q

MYCOBACTERUIM AVIUM
menciona las principales enfermedades de este complejo

A

-Pulmonar localizada: hombre con antecedente de tabaquismo o enfermedad pulmonar vase
-Nodulo pulmonar solitari: infrecuente
-Enfermedad diseminada: principalmente con SIDA, formas graves en conteos menoresa<10cel/mm3 de CD4

How well did you know this?
1
Not at all
2
3
4
5
Perfectly
33
Q

STREPTOCOCCUS PYOGENES
Menciona los componentes de su estructura y sus características principales

A

Es el más importante de los estreptococos del grupo A de Lancefield. Coco gram + anaerobio facultativo que crece formando cadenas.
Catalasa -, L-pirrolidonil arilamidasa (PYR) +.
- Proteína M: adenina antifagocítica, degrada C3b.
- Exotoxinas pirógenas termilábiles: producción del exantema escarlatiforme (SpeA, SpeB, SpeC, SpeF), actúa como superantígeno y provoca el síndrome de choque tóxico.
- Estreptolisina S: responsable de hemoptisis β.
- Estreptolisina O: los anticuerpos antiestreptolisina O (ASLO) demuestran infección reciente.

Es la causa más frecuente de faringitis bacteriana (5 - 15 años)

How well did you know this?
1
Not at all
2
3
4
5
Perfectly
34
Q

STAPHYLOCOCCUS AUREUS
¿Qué enfermedades produce este agente?

A

CUTÁNEAS: Impétigo, folliculitis, forúnculo, ántrax (carbunco), infección de heridas.
MEDIADAS POR TOXINAS: intoxicación alimentaria, síndrome de la piel escaldada (enfermedad de Ritter), síndrome de choque tóxico.
OTRAS: artritis séptica, bacteremia, endocarditis, osteomielitis, neumonía, empiema.

How well did you know this?
1
Not at all
2
3
4
5
Perfectly
35
Q

STAPHYLOCOCCUS AUREUS
Menciona los criterios diagnósticos para el síndrome de choque tóxico

A
  • Fiebre
  • Hipotensión
  • Erupción macular difusa
  • Involucramiento de 3 de los siguientes órganos: hígado, sangre, riñón, membranas mucosas, tubo digestivo, músculos, SNC.
  • Resultado negativo en aerologías de sarampión, leptospirosis y fiebre moteada de las montañas rocosas.
    Hemocultivo o cultivo de LCR negativos para otros organismos además de S. aureus.
    TODOS DEBEN CUMPLIRSE.
How well did you know this?
1
Not at all
2
3
4
5
Perfectly
36
Q

STAPHYLOCOCCUS AUREUS
Menciona los principios de tratamiento en las diferentes enfermedades que causa

A
  • Abscesos cutáneos: drenaje
  • Enfermedades cutáneas y de tejidos blandos: sensible a meticiclina (dicloxacilina, penicilinas resistentes a peniclinasas, cefalosporinas); SAMR (ceftrarolina, trimethoprim-sulfametoxazol, doxiciclina, minociclina).
  • Infecciones invasivas por SAMR: vancomicina, linezolid.
  • Osteomielitis por SAMR: rifampicina
  • Síndrome de choque tóxico: ATB + drenaje + líquidos IV, puede ser necesario vasopresores e inmunoglobulina IV.
How well did you know this?
1
Not at all
2
3
4
5
Perfectly
37
Q

STAPHYLOCOCCUS EPIDERMIDIS
Menciona sus características principales

A

Coco gram+, catalasa+ y coagulasa-; expresa la toxina δ.
Parte de las principales causas de endocarditis en las prótesis valvulares.
Resistente a meticilina.

How well did you know this?
1
Not at all
2
3
4
5
Perfectly
38
Q

STAPHYLOCOCCUS EPIDERMIDIS
¿Cuáles son las enfermedades que produce?

A
  • Bacteremia
  • Endocarditis
  • Infección de heridas quirúrgicas
  • Infecciones del tracto urinario
  • Oportunistas: catéteres, anastomosis, prótesis, dispositivos de diálisis peritoneal
How well did you know this?
1
Not at all
2
3
4
5
Perfectly
39
Q

STREPTOCOCCUS PYOGENES
Menciona el diagnóstico y describe los criterios de Centor

A

Crecimiento óptimo en agar sangre (estándar de oro) enriquecido, presentando patrón de hemolisis β.

Criterios para dx de sindrome de choque tóxico:
- Aislamiento de un estreptococo del grupo A
- Hipotensión
- Dos de las siguientes manifestaciones: disfunción renal, involucramiento hepático, erupción mácula eritematosa, coagulopatía, necrosis de tejidos blandos, síndrome de dificultad respiratoria del adulto.

Criterios de Centor:
- Temperatura >38° (+1 punto)
- Ausencia de tos (+1 punto)
- Adenopatías dolorosas cervicales anteriores (+1 punto)
- Hipertrofia o exudado amigdalino (+1 punto)
- 3 - 14 años (+ 1 punto)
- >44 años (-1 punto)

≤2 puntos: no se toma cultivo ni se da ATB
3 - 5 puntos: iniciar ATB, valorar

How well did you know this?
1
Not at all
2
3
4
5
Perfectly
40
Q

STREPTOCOCCUS PYOGENES
Describe el tratamiento

A

Sensible a bacitracina y resistente a optoquinona.
Primera línea faringitis
- Niños <27 kg: penicilina G benzatínica 600 000 UI IM
- Niños >27 kg: penicilina G benzatínica 1200 000 UI IM

Alternativo:
- Bencilpenicilina procaínica con bencilpenicilina cristalina (penicilina procaínica) por 3 dosis (una c/24h) + 1 dosis de penicilina benzatínica el 4to día.
- penicilina V o amoxi-clav por 10 días

Segunda línea o hipersensibilidad a penicilinas: eritromicina, trimethoprim-sulfametoxazol y cefalosporinas de 1ra gen.

Medidas no farmacológicas: elevación de la cabecera con aspiración gentil de secesión nasal con perilla, reposo, inhalación de vapor, dieta con incremento de aporte de líquidos.

How well did you know this?
1
Not at all
2
3
4
5
Perfectly
41
Q

STREPTOCOCCUS PYOGENES
¿Qué enfermedades produce?

A

SUPURATIVAS:
- Faringitis
- Escarlatina
- Pioderma
- Erisipela
- Celulitis
- Fascitis necrosante
- Síndrome de choque tóxico
- Septicemia puerperal
- Linfangitis
- Neumonía

NO SUPURATIVAS:
- Fiebre reumática
- Glomerulonefriis posestreptocócica aguda

How well did you know this?
1
Not at all
2
3
4
5
Perfectly
42
Q

STREPTOCOCCUS PNEUMONIAE
Describe las características de su estructura y sus factores de virulencia

A

Coco gram+ encapsulado que crece típicamente en pares o cadenas cortas, en apariencia de lanceta; anaerobio facultativo; coloniza bucofaringe y es capas de diseminarse a pulmones, oído medio y cerebro.
- Factores que intervienen en la colonización y migración: adhesivas proteicas de superficie, proteasa de IgA secretora y neumolisina.
- Factores que intervienen en la destrucción tisular: ácido teicoico y fragmentos de proteoglucanos.
- Factores que intervienen en la supervivencia frente a fagocitosis: cápsula y neumolisina.

How well did you know this?
1
Not at all
2
3
4
5
Perfectly
43
Q

E.COLI
describe brevemente sus caracteristicas microbiológicas

A

Familia enterobacteriae (salmonella, shigella, yersinia, proteus, citrobacter, enterobacter etc)
-Bacilo gram negativo con antígeno enterobacteriano
-Anaerobios facultativos, catalasa+, oxidasa-, fermenta glucosa, oxida nitratos
-Lipopolisacárido LPS termoestable: Principal antígeno de pared celular (polisacárido externo somático O, núcleo polisacárido (agcomun) lipido A (endotox))
-Flora bacteriana normal del humano
-Adhesinas (fimbrias, pilli) eliminan efecto de arrastre por miccion o peristalsis

How well did you know this?
1
Not at all
2
3
4
5
Perfectly
44
Q

E.COLI
Menciona las enfermedades básicas que produce y su via de transmision

A

Mayoria de infecciones son endogenas
Gastroenteritis por cepas exogenas
-Via: agua y alimentos contaminados

-BACTEREMIA: bacilo G- mas aislado en USA, inicia en vias urinarias o tracto digestivo. mortalidad elevada en inmunocompoometidos o focos abdominales/nSNC

-IVU: bacteria mas frecuente, cistitis mas comun, puede axtender a pielo o prostatitis

-MENINGITIS NEONATAL: 75% cepas con AG K1, mayoria de infecciones SNC en <1mes

-GASTROENTERITIS POR CEPAS: “entero”patogena (EPEC), toxigena (ETEC), hemorragica (EHEC), invasiva (EIEC), agregativa (EAEC), difusamente adherente (DAEC).

How well did you know this?
1
Not at all
2
3
4
5
Perfectly
45
Q

E.COLI
¿cuales son las gastroenteritis que causan cada una de las cepas?

A

-EPEC (delgado): diarrea infantil en paises subdesarrollados, acuosa, vomito heces sin sangre. destrullen microvellosidades de los enterocitos

  • ETEC(delgado): diarrea del viajero, acuosa vomito, espasmos abdominales, nausea (agua contaminada con fecal, incba 1-2d, dura 3-4d). enterotoxinas termoestables aumentan AMPc estimulando hipersecresion de CL e hipoabsorcion de na y cl, deplesion de celulas caliciformes
  • EHEC: (grueso): D. acuosa al inicio que se vuelve sanguinolenta, espasmo ab, progresa a SxHemoUre, productos mal procesados. Toxinas shiga sxmalabsorcion

-EIEC (grueso): paises subdes. fiebre, diarrea–>disenteria. incuba 1-3d dura 7-10d

-EAEC (delgado): viajero o infantil en paises sub, D.acuosa, vomito, deshidratacion, febricula, malnutricion

-DAEC (delgado) diarrea persitentes en niños mayores a 18 meses

How well did you know this?
1
Not at all
2
3
4
5
Perfectly
46
Q

E.COLI
cual es el tratamiento en las gastroenteritis generadas por este patogeno

A

TTO primario niños: reposicion y mantenimiento de liquidos y electrolitos, si se requiere atb en menores de 5 años se recomienda azitromicina para ETEC, cifexime o azitro para ECEH. no ATB en EAEC.
TTO adultos reposicion y mantenimiento de liquidos. ATB cipro por 3 dias

How well did you know this?
1
Not at all
2
3
4
5
Perfectly
47
Q

SHIGELLA
menciona sus características microbiológicas principales

A

conformado por Sdysenteriae (produce exotoxina shiga, enfermedades graves), S. flexneri (mas frecuente de shigelosis), S. boydii (muerte en niños desnutridos), S.sonnei shigelosis en paises industrializados )
-Bacilo G-, anaerobio facultativo, fermentados, oxidasa negativo
-invade celilas M en placas de peyer, induce apoptosis, altamente infecciosos
-Transmite fecal-oral

How well did you know this?
1
Not at all
2
3
4
5
Perfectly
48
Q

SHIGELLA
menciona la accion de la toxina shiga

A

Pertenece a Sdysenteriae
-1 subunidad A (afecta sub-60s, altera sintesis proteica), 5 subunidades B (union)
-Daño epitelio GI (colitis hemorragica)
-Puede causar daño glomerular (SxHemoUre)

How well did you know this?
1
Not at all
2
3
4
5
Perfectly
49
Q

SHIGELLA
menciona las enferemedades porducidas por esta bacteria

A

-GASTROENTERITIS (shigelosis). incuba 1-3 dias con fiebre, escalofrio, mialgia, anorexia. diarrea acuosa en delgado o sangre en colon, RESUELVE ESPONTANEO

-DISENTERIA: forma grave, sangre moco y pus, espasmo abdominal, tenesmo, abseso en criptas y ulceras en mucosa

-COMPLICACIONES: convulsiones febriles, baceremia, sintomas neurologicos, obstruccion, megacolon, sxhemoure.

-SX postinfecciosos: artritis reactiva

How well did you know this?
1
Not at all
2
3
4
5
Perfectly
50
Q

SHIGELLA
menciona metodo diagnostico y tratamiento

A

Clinica ya mencionada o se puede agregar: malnutricion proteico-energetica (kwshiorkor)
-Aislamiento en heces: agar MacConkey, agar eosina-azul de metileno, xilosa-lisina-deoxicolato
TTO atb: en adultos ciprofloxacino 1ra eleccion, trimetoprim-sulfametoxazol alternativa. Niños: azitro o ceftria 1ra, cefixima, cipro, trimetoprim-sulfa alternativa

How well did you know this?
1
Not at all
2
3
4
5
Perfectly
51
Q

STREPTOCOCCUS PNEUMONIAE
¿Qué enfermedades produce y cómo se diagnóstica?

A
  • NEUMONÍA: escalofríos, fiebre sostenida (39-41°), tos, dolor pleurítico y consolidación de lóbulos inferiores; 25% presentan derrame pleural.
  • SINUSITIS Y OTITIS MEDIA: precede de una infección de vías respiratorias inferiores, se obstruyen senos nasales y conducto auditivo.
  • MENINGITIS: cefalea, fiebre y septicemia.
  • BACTEREMIA: px con meningitis, puede ser fulminante en combinación con asplenia.

DX: clínico + microscopio + cultivo (agar sangre)

How well did you know this?
1
Not at all
2
3
4
5
Perfectly
52
Q

STREPTOCOCCUS PNEUMONIAE
Menciona el tratamiento segun la gravedad de NAC

A

Niños de 3m a 18a
- NAC no grave: ambulatorio, amoxicilina VO
- NAC grave: hospitalario, penicilina G IV. Alternativa: cestriaxona o cefotaxima.

Adutltos:
- NAC leve (CURB 65 0-1): amoxicilina.
- NAC moderada (CURB 65 2): quinolona respiratoria VO o IV; cefalosporina de 3ra generación + macrolido o amoxi/clav + macrolido.
- NAC severa (CURB 65 3-5): igual que moderada excepto si esta en UCI; β-láctamico IV + macrolido IV o β-lactámico IV + quinolona IV.

How well did you know this?
1
Not at all
2
3
4
5
Perfectly
53
Q

STREPTOCOCCUS AGALACTIAE
Menciona su estructura, comportamiento y enfermedades que causa

A

Coco gram+ anaerobio facultativo catalasa-, resistente a bacitracina y optoquinoa. Coloniza tractos respiratorio y genitourinario.
Es la causa más frecuente de septicemia y meningitis neonatales.

Produce enfermedades neonatales de inicio precoz (neumonía, meningitis y septicemia), tardío (bacteremia y meningitis entre 1 y 3 sec del nacimiento); infecciones en mujeres gestantes (genitourinarias) e infecciones en adultos (bacteremia, neumonía, óseas, articulares, cutáneas y de tejidos blandos).
β-lactámicos como tx de elección.

How well did you know this?
1
Not at all
2
3
4
5
Perfectly
54
Q

LISTERIA MONOCYTOGENES
Describe su estructura

A

Bacilo gram+ pequeño no ramificado, anaerobio facultativo, prolifera en temp de 1-45°; crece en pares o cadenas, presenta hemoptisis β; es intracelular facultativo, puede crecer en macrófagos, c. epiteliales y fibroblastos en cultivo.

How well did you know this?
1
Not at all
2
3
4
5
Perfectly
55
Q

LISTERIA MONOCYTOGENES
Describe su comportamiento y las enfermedades que produce

A

Se limita a neonatos, ancianos, gestantes e inmunodeprimidos. Se asocia a alimentos contaminados. Es la causa más común de meningitis bacteriana en px con linfoma, receptores de trasplante y tratados con corticoides por cualquier razón.
Produce granulomatosis infantiséptica, enfermedad neonatal de comienzo tardío (meningitis, meningoencefalitis y septicemia); enfermedad pseudogripal acompañada o no de gastroenteritis en adultos; romboencefalitis listerióstica (encefalitis que involucra al tallo encefálico y afecta adultos sanos.

How well did you know this?
1
Not at all
2
3
4
5
Perfectly
56
Q

LISTERIA MONOCYTOGENES
Describe diagnóstico y tratamiento

A

Cultivos de sangre y LCR en agar semisólido.

El tx de elección es ampicilina o amoxicilina.
- Bacteremia son involucramiento de SNC: 2 sem.
- Meningitis: al menos 3 sem.
- Endocartidis o absceso cerebral: al menos 6 sem.
- Px con alteración de la función de linfos T agregar gentamicina.
- Hipersensibilidad a penicilina: trimetoprim-sulfametoxazol.

How well did you know this?
1
Not at all
2
3
4
5
Perfectly
57
Q

NOCARDIA
¿Qué es la nocardiosis y cuáles son las especies responsables?

A

Es un grupo de infecciones causadas por actinomicetos aerobios y de membrana gram+ encontrados en el suelo, capaces de invadir pulmones, tejidos blandos y SNC.
Las especies responsables son N.asteroides (pulmonar), N.brasiliens (micetoma) y N.otitidiscaviarum.

How well did you know this?
1
Not at all
2
3
4
5
Perfectly
58
Q

NOCARDIA
Describe diagnóstico y tratamiento

A

Indice elevado de sospecha clínica, confirmación por tinción bacteriológica y cultivo de esputo, lesiones cutáneas o material purulento de abscesos cerebrales, pulmonares o derrames pleurales.
Tinción: Ziehl-Nielsen.
Rx y TAC: infiltrados difusos y cavilaciones en caso de involucramiento pulmonar.

TRATAMIENTO
-Infecciones cutáneas: trimetoprim-sulfametoxazol; en infecciones linfocutáneas el tx puede extenderse 2-4 meses.
-Micetoma: 6-12 meses.
-Infecciones sistémicas sin involucrar SNC: trimetoprim-sulfametoxazol + imipenem.
-Involucramiento multiorgánico: agregar amikacina.
-Abcesos: drenaje quirúrgico y tx sistémico por 12 meses.
-Inmunosuprimidos: atb por 12 meses.

How well did you know this?
1
Not at all
2
3
4
5
Perfectly
59
Q

ACTINOMYCES
¿Qué es la actinomicosis, cómo se diagnóstica y maneja?

A

Gram+, no especulada, anaeróbia facultativa o estricta, considerada comensal en orofaringe, trato gastrointestinal y genitourinario femenino.
Las infecciones son invasivas raras, granulomatosas subagudas o crónicas.
Las manifestaciones varían con el sitio de infección, el diagnóstico se confirma con histopatológico de biopsia de tejido infectado.

Tratamiento: β-lactámicos (penicilina G o V + amoxicilina) por 6-12 meses.
Alternativas: ceftriaxona, eritromicina, clindamicina y doxicilcina.

How well did you know this?
1
Not at all
2
3
4
5
Perfectly
60
Q

TROPHERYMA WHIPPLEI
Describe qué es la enfermedad de whipplei (EW), su comportamiento, diagnóstico y manejo

A

Es una infección crónica poco frecuente, se manifiesta con síntomas gastrointestinales y sistémicos (fiebre, artralgias y alteraciones neurológicas).

T. whipplei esta presente en trabajadores expuestos a aguas residuales, granjeros y agricultores, el 90% de afectados son hombres blancos de 40-50 años.

EW tiene 4 síntomas cardinales: artralgias, diarrea crónica, dolor abdominal tipo cólico y pérdida de peso.
El estudio de elección es la endoscopia con toma de biopsias gastrointestinales.
Se confirma cuando la biopsia muestra macrófagos positivos con tinción de ácido periódico de Schiff; con el microorganismo en tinción de inmunohistoquímica con antisueros específicos para T. whipplei o por PCR.

El tx de elección es ceftriaxona o penicilina G IV por 2 sem en fase inicial (4 sem si endocarditis), seguido de trimetoprim sulfametoxazol por 12 semanas.

How well did you know this?
1
Not at all
2
3
4
5
Perfectly
61
Q

Campylobacter (jejuni, coli, upsaliensisis y fetus) características

A

bacilos gram –, móviles en forma de coma su principal antígeno es el LPS.
infecciones principalmente zoonóticas (aves de corral, alimentos contaminados etc.)

How well did you know this?
1
Not at all
2
3
4
5
Perfectly
62
Q

Campylobacter (jejuni, coli, upsaliensisis y fetus) que produce

A

Enteritis aguda con diarrea (<=7 días) (≥10 evacuaciones/día en el periodo de mayor intensidad), malestar general, fiebre y dolor abdominal.

How well did you know this?
1
Not at all
2
3
4
5
Perfectly
63
Q

Complicaciones de la infección por Campylobacter:

A

hemorragia GI masiva, megacolon tóxico, infección del tracto biliar (provocando hepatitis obstructiva, colecistitis o pancreatitis), peritonitis, ruptura esplénica y exacerbaciones de la colitis inflamatoria. Entre las complicaciones extraintestinales se encuentran la bacteriemia, meningitis, endocarditis, osteomielitis artritis purulenta, reactiva (HLA-B27)GUILLIAN-BARRE.
uveítis, síndrome hemolítico urémico, eritema nodoso, encefalitis, carditis y anemia hemolítica.

How well did you know this?
1
Not at all
2
3
4
5
Perfectly
64
Q

¿A qué enfermedad autoinmune está estrechamente asociada la infección por C. jejuni y C. upsaliensis?

A

SX de Guillain-Barré debido a la reactividad antigénica cruzada entre los oligosacáridos de la cápsula bacteriana (de C. jejuni y C. upsaliensis) y los glucoesfingolípidos de la superficie de los tejidos neuronales.

How well did you know this?
1
Not at all
2
3
4
5
Perfectly
65
Q

POXVIRUS
cual es la composición de los poxvirus

A

familia pocviridaese, DNA bicatenario
inlcuye: ortopoxvirus (responsable de la viruela) y molluscipoxivirus (responsable del molusco contagioso)

How well did you know this?
1
Not at all
2
3
4
5
Perfectly
66
Q

POXVIRUS
cual es la instauracion del molusco contagioso (mulluscipoxivirus)

A

replicacion viral de las capas profundas de la dermis, llegando a las superficiales, incubacion de 2-7 semanas llegando hasta los 6 meses

How well did you know this?
1
Not at all
2
3
4
5
Perfectly
67
Q

POXVIRUS
cuales son las inclusiones caracteristicas del molusco contagioso

A

cuerpos de Henderson-Paterson, se forman en el estrato espinoso y crecen y migran a la superficie

How well did you know this?
1
Not at all
2
3
4
5
Perfectly
68
Q

POXVIRUS
cual es el medio de transmisión del molusco contagioso

A

ocurre con disrupción en la piel, relacionado con fómites en toallas compartidas y en adultos genitales es relacionado con contacto sexual

How well did you know this?
1
Not at all
2
3
4
5
Perfectly
69
Q

POXVIRUS
cual es la lesión característica de molusco contagioso

A

pápula pequeña que madura y se convierte en nódulo de 2-5mm, liso con forma de domo y coloración PERLADA, puede haber salida de material blanquecino. Suelenm desarrollarse de 1-20 lesiones (pueden verse 100tos) y diseminarse segun la mecanica del rascado.

How well did you know this?
1
Not at all
2
3
4
5
Perfectly
70
Q

POXVIRUS
cual es la presentación clásica del molusco contagioso

A

en niños de 1-4a se presentan en tronco y extremidades proximales
en adultos se presentan en tronco y area pubica

How well did you know this?
1
Not at all
2
3
4
5
Perfectly
71
Q

POXVIRUS
cual es la duracion de la infeccion por molusco contagioso

A

inmunocompetentes, es autolimitada y dura 2 meses (lesion individual) la resolucion puede tardar de 6-9 meses

How well did you know this?
1
Not at all
2
3
4
5
Perfectly
72
Q

POXIVIRUS
cuales son las indicaciones de tto en molusco y cuales son

A

puede emplearse en afectacion facial o multiple por razones esteticas.
-crioterapia, curetaje, podofilina, cantaridina, yoduro, tretinoina, cidovofir 3%

How well did you know this?
1
Not at all
2
3
4
5
Perfectly
73
Q

VIH
describe la composición del virus del VIH

A

lentivirus ARN de cadena positiva de la familia RETROVIRIDAE con un virión esférico de núcleo electrodenso rodeado por envoltorio lipídico con glucoproteínas gp120 y gp41
contiene enzimas viricas: proteasa, transcriptasa inversa e integrasa.

How well did you know this?
1
Not at all
2
3
4
5
Perfectly
74
Q

Virus “Varicela Zoster”

A

Alfa herpes virus
Familia: Herpesviridae
Subfamilia: alphaherpesvirinae
Envoltura lípidica rodea a una nucleocapside de simetría icosaédrica
ADN bicatenario

How well did you know this?
1
Not at all
2
3
4
5
Perfectly
75
Q

VIH
que proteina es responsable de la deficiencia de CD4

A

proteina NEF, reduce exprecion de CD4 y MHC1, alterando vias de señalización de linfocitos T regulando la toxicidad virica elevando carga viral

How well did you know this?
1
Not at all
2
3
4
5
Perfectly
76
Q

Que causa el virus “Varicela Zoster”

A

Primoinfección: Varicela
y Herpes Zoster

How well did you know this?
1
Not at all
2
3
4
5
Perfectly
77
Q

Como se transmite el virus “Varicela Zoster”

A

Contacto directo a traves de vesículas o
Aerosoles de secreciones respiratorias

How well did you know this?
1
Not at all
2
3
4
5
Perfectly
78
Q

Después de la primoinfección donde que permanece latente el virus “Varicela Zoster”

A

Neuronas de los ganglios de las raíces dorsales, ganglios autonomos y nervios craneales

How well did you know this?
1
Not at all
2
3
4
5
Perfectly
79
Q

VIH
cual es la clasificacion por carga viral A

A

A= asintomatica, HIV agudo, linfadenopatia generalizada progresiva
1: >= 500/ml

How well did you know this?
1
Not at all
2
3
4
5
Perfectly
80
Q

¿Qué es el Herpes Zóster y como se manifiesta?

A

Reactivación espontanea del VVZ
Manifiesta: exantema vesicular doloroso se localiza sobre un dermatoma, produciendo una disfunción neuroquímica responsable del dolor radicular que puede ser moderada o intensa

How well did you know this?
1
Not at all
2
3
4
5
Perfectly
81
Q

VIH
cual es la clasificación para espectro B

A

son condiciones atribuidas a la infeccion por VIH o que indican defecto en inmunidad celular
-candidiasis orofaringea, vulvovaginal persistente o recurrente, displasia cervical moderada-severa, fiebre, diarrea >1mes, leucoplaquia vellosa oral, herpes >=2 veces,PTI,

How well did you know this?
1
Not at all
2
3
4
5
Perfectly
82
Q

VIH
cual es la clasificacion para el espectro C

A

CONDICIONES INDICADORAS DE SIDA
- candidiosis bronquial, traqueal o pulmonar, CACU invasivo, coccidiomicosis diseminada, criptococosis, herpes >1 mes, kaposi, burkit, pneumocystis jiroveci, neumonia recurrente, sakmunelos septica, toxoplasmosis cerebral etc

How well did you know this?
1
Not at all
2
3
4
5
Perfectly
83
Q

VIH
cual es la clasificacion del espectro por conteo de linfocitos

A

1: >=500/mcl
2: 200-499/mcl
3: <200/mcl
cada uno de estos numeros se combina segun su sintomatologia con las clasificaciones ABC y se establese un espectro espcifico

How well did you know this?
1
Not at all
2
3
4
5
Perfectly
84
Q

VIH
cual es la manifestacion clinica de la infeccion aguda por VIH

A

sindrome retroviral agudo: similar a mononucleosis, fiebre, odnofagia, adenopatia, cefalea, erupcion como roseola
aparece 3-6 semanas despues de la infeccion, resuelve 2-4 sem

How well did you know this?
1
Not at all
2
3
4
5
Perfectly
85
Q

VIH
menciona los hongos oportunistas en SIDA

A

candida, cryptococcus, coccidioides, histoplasma

How well did you know this?
1
Not at all
2
3
4
5
Perfectly
86
Q

VIH
menciona las bacterias oportinistas de SIDA

A

micobacterium avium-tuberculosis, nocardia, salmonella

How well did you know this?
1
Not at all
2
3
4
5
Perfectly
87
Q

VIH
menciona los virus oportunistas de SIDA

A

CMV, VHS, varicela z, virus JC

How well did you know this?
1
Not at all
2
3
4
5
Perfectly
88
Q

Periodo de incubación de la hepatitis A

A

15 a 50 días.
28 días en promedio

How well did you know this?
1
Not at all
2
3
4
5
Perfectly
89
Q

VIH
menciona protozoarios y helmintos oportunistas de SIDA

A

cryptosporidium, cystospora, pneumocystis jirovecii, toxoplasma gondii, strongoloides

How well did you know this?
1
Not at all
2
3
4
5
Perfectly
90
Q

Prevalencia de hepatitis A en México

A

90%

How well did you know this?
1
Not at all
2
3
4
5
Perfectly
91
Q

VIH
menciona las neoplasias asociadas a SIDA

A

1.sarcoma de kaposi en menores de 60 (es la mas comun)
2.LNHKN celulas B grado alto
3.Linfoma primario de SNC
4. LBurkitt
5. LNHKN indiferenciado
6. Linfima inmunoblastico
7. linfomas cavidades corporales
8. CACU invasor

How well did you know this?
1
Not at all
2
3
4
5
Perfectly
92
Q

Permanencia viable del virus de la hepatitis A

A

Por días o semanas en mariscos, agua, tierra o sedimento marino.
Permanece semanas o meses en el ambiente.
Puede sobrevivir hasta 4 hrs en las manos

How well did you know this?
1
Not at all
2
3
4
5
Perfectly
93
Q

VIH
Cual es la manera correcta de diagnosticar la infección

A

2 pruebas presuntivas (ELISA, quimiolumi, aglutinación, rápida) positivas, debe ser la misma preferentemente
1 prueba confirmatoria: WB, inmunofluorescencia indirecta, radioinmunoprecipitacion, carga viral por PCR, amplificación de ácidos nucleicos

How well did you know this?
1
Not at all
2
3
4
5
Perfectly
94
Q

Personas con alto riesgo de adquirir la hepatitis A

A

Consumo de agua contaminada, HSH, usuarios de drogas iv, personas con trastornos de coagulación que requieren la administración de factores de coagulación, contcatos cercanos de personas infectadas.

How well did you know this?
1
Not at all
2
3
4
5
Perfectly
95
Q

VIH
como se define TAR segun GPC

A

combinacion de 3 o + farmacos de alta potencia utilizados para el manejo de infeccion por VIH

How well did you know this?
1
Not at all
2
3
4
5
Perfectly
96
Q

Cuadro clínicio Hepatitis A

A

-Asintomatico y subclínica. Mayoría de los casos 3/4 de la población adulta presenta síntomas, 90% <2 años son asintomaticos.

Edad determinante de la severidad.
Síntomas: malestar general 80%, naúsea 87%, vómito 47%, anorexia, fiebre 76%, dolor en hipocondrio derecho 37%.

3-7 días después del inicio de los síntomas: coluria, acolia, ictericia y prurito.

How well did you know this?
1
Not at all
2
3
4
5
Perfectly
97
Q

VIH
que significa esquema de inicio, de segunda linea y de rescate para TAR

A

INICIO: TAR en un paciente que NUNCA ha tomado ARV
SEGUNDA LINEA: TAR en un paciente que falla con un primer esquema de ARV
RESCATE: TAR en un paciente con resistencia a dos o mas ARV independientemente del numero de esquemas previos

How well did you know this?
1
Not at all
2
3
4
5
Perfectly
98
Q

Exploración física en infección por virus de la hepatitis A

A

Ictericia, hepatomegalia 87%, esplenomegalia 9%, exantema 3%, edema 2%, petequias 2%, arritmias cardíacas 2%

How well did you know this?
1
Not at all
2
3
4
5
Perfectly
99
Q

VIH
al iniciar con ABC, que estudio debe solicitarse al paciente

A

HLA-B 5701, si la prueba resulta positiva no debe administrarse ABC

How well did you know this?
1
Not at all
2
3
4
5
Perfectly
100
Q

Hallazgos bioquímicos en la infección por virus del hepatitis A

A

Elevación de la bilirrubina total a expensas de la directa, aminotransferasas séricas superior a 1000 UI/dl

How well did you know this?
1
Not at all
2
3
4
5
Perfectly
101
Q

VIH
por que se prefieren los INSTI a los IP en ARV

A

INSTI tiene mayor tolerancia respecto a IP, por lo que es menos descontinuado

How well did you know this?
1
Not at all
2
3
4
5
Perfectly
102
Q

VIH
en quienes esta indicado el TAR

A

todos los adultos con VIH independientemente del conteo de CD4 y de la existencia de sintomas

How well did you know this?
1
Not at all
2
3
4
5
Perfectly
103
Q

Diagsnotico del virus hepatitis A

A

Anticuerpos anti-VHA IgM.
Detectan a partir de los 15 días de la aparición de los síntomas y persisten hasta 6 meses después de la infección

How well did you know this?
1
Not at all
2
3
4
5
Perfectly
104
Q

Tratamiento en virus del hepatitis tipo A

A

Sosten y se debe evitar el uso del paracetamol

How well did you know this?
1
Not at all
2
3
4
5
Perfectly
105
Q

Profilaxis post-exposición para hepatitis virus A

A

Vacuna o inmunoglobulina

Dentro de las primeras 2 semanas del incio de los síntomas o del contacto con el caso índice.

Pacientes inmunosuprimidos o con hepatopatía crónica se prefiere inmunoglobulina.

How well did you know this?
1
Not at all
2
3
4
5
Perfectly
106
Q

VIH
que condiciones indican TAR de la manera mas rapida posible

A

CD4<200, embarazo, nefropatia asociada a VIH, SIDA, deterioro cognitivo, primoinfeccion o sindrome retroviral agudo

How well did you know this?
1
Not at all
2
3
4
5
Perfectly
107
Q

Como se presenta la hepatitis fulminante por virus del hepatitis A

A

Ictericia, encefalopatía, deterioro de la función hepática y prolongación de los tiempos de coagulación.

How well did you know this?
1
Not at all
2
3
4
5
Perfectly
108
Q

VIH
cuales son las indicaciones de indicar TAF si no se indico dentro del TAR inicial

A

ERC, disminucion de funcion renal, probabilidad de fractura a 10 años >=20%, enfermedad osea

How well did you know this?
1
Not at all
2
3
4
5
Perfectly
109
Q

Que puede desencadenar la infección por VHA

A

Presentación de hepatitis autoinmune.

How well did you know this?
1
Not at all
2
3
4
5
Perfectly
110
Q

VIH
cuando tomar CV y conteo de CD4 previo al TAR

A

ambos al inicio, si se difiere TAR por mas de 6 meses, se repite CV antes de iniciar, y el CD4 cada 4-6 meses

How well did you know this?
1
Not at all
2
3
4
5
Perfectly
111
Q

Medidas preventivas de VHA

A

Vacunación
lavar frutas y verduras, tomar agua hervida o clorada, lavado de manos antes de comer y después de ir al baño, evitar comer alimentos crudos.

How well did you know this?
1
Not at all
2
3
4
5
Perfectly
112
Q

VIH
cuando tomar CV y CD4 despues de iniciar TAR

A

CV: 2m despues de iniciar TAR, posterior a 6to y 10 mes, posterior cada 4-6m los primeros 2 años
CD4: 2-6m despues de iniciar TAR, despues cada 4-6m los primeros 2 años

How well did you know this?
1
Not at all
2
3
4
5
Perfectly
113
Q

Virus de la hepatitis. C

A

Virus RNA
Familia: flaviviridae
Genero: hepcivirus
Replica: en los hepatocitos, al no estar directamente citopático se asocia a una infección persistente.

How well did you know this?
1
Not at all
2
3
4
5
Perfectly
114
Q

VIH
cuando tomar CV y CD4 si se modifica el TAR

A

CV: 2m despues de iniciar TAR, posterior a 6to y 10 mes, posteri
CD4: 2-6m despues de iniciar TAR, despues cada 4-6m los primeros 2 años

How well did you know this?
1
Not at all
2
3
4
5
Perfectly
115
Q

Genotipos en VHC

A

Existen 6 genotipos y 50 subtipos

Genotipo 1 es responsable 70-75%
México hay prevalencia 1-1.9%

How well did you know this?
1
Not at all
2
3
4
5
Perfectly
116
Q

VIH
si el conteo de CD4 es menor a 300, cada cuanto se debe tomar CV y CD4

A

ambos cada 4-6 meses

How well did you know this?
1
Not at all
2
3
4
5
Perfectly
117
Q

Principal causa de hepatopatía crónica en el mundo

A

Infección del VHC
27% casos de cirrosis
25% de los casos hepatocarcinoma

How well did you know this?
1
Not at all
2
3
4
5
Perfectly
118
Q

VIH
en paciente con al menos 2años de TAR y CV indetectable con CD4 >=300 cada cuanto debe tomarse CV y CD4

A

CV: cada 6 meses
CD4: cada 12 meses, si el conteo es >500 es opcional en indetectables

How well did you know this?
1
Not at all
2
3
4
5
Perfectly
119
Q

Evolución clinica infección por VHC

A

55-85% curso crónico
5-25% desarrollan cirrosis hepática lapso de 20-25 años
Riesgo aprox. 1-2% por año de aparición de carcinoma hepatocelular

How well did you know this?
1
Not at all
2
3
4
5
Perfectly
120
Q

VIH
en pacientes con TAR y CV detectable, cada cuanto se deben tomar CV y CD4

A

CV: lo antes posible, preferentemente <4semanas
CD4: cada 4-6 meses

How well did you know this?
1
Not at all
2
3
4
5
Perfectly
121
Q

Poblaciones de riesgo VHC

A

Receptores de sangre o tejidos, pacientes con hemodiálisis, trabajadores de la salud, elevación inexplicable de aminotransferasas, usuarios de drogas IV, personas que viven con VIH y VHB, relaciones sexuales de riesgo, tatuajes, perforaciones, reutilización de agujas, hombres que tienen sexo con hombres, vertical (madre-hijo), compartir objetos punzo-cortantes de uso personal, presos.

How well did you know this?
1
Not at all
2
3
4
5
Perfectly
122
Q

VIH
en falla virologica, que accion se toma con CV entre 50-200 copias

A

se considera viremia de bajo nivel
-vigilancia cada 6 meses, si persiste por 12 meses se deriva a especialista

How well did you know this?
1
Not at all
2
3
4
5
Perfectly
123
Q

Factores asociados a mayor progresión de fibrosis hepática en VHC

A

Hombres
Mayores de 40 años
Afroamericanos
IMC >25
Tabaquismo
Consumo de alcohol >50g/día
Coinfección con VIH y/o VHB
Resistencia a la insulina
Genotipo 3 de VHC

How well did you know this?
1
Not at all
2
3
4
5
Perfectly
124
Q

VIH
en falla viral, que accion se toma con CV entre 200-1000 copias

A

-considerar cambio farmacologico empirico
-SE RECOMIENDA GENOTIPO VIRAL SI ES >200 copias

How well did you know this?
1
Not at all
2
3
4
5
Perfectly
125
Q

Cuadro clínico en VHC

A

Asintomaticos
Una parte refiere fatiga o ataque al estado general.
Manifestaciones extrahepaticas:
crioglobulinemia, porfiria cutánea tarda, liquen plano.
25% aminotransferasas normales
Resto de pacientes presenta elevación por encima del límite superior normal.

How well did you know this?
1
Not at all
2
3
4
5
Perfectly
126
Q

VIH
en falla viral, en pacientes con >=1000 copias, que accion se debe tomar

A

-realizar genotipo viral
-identificar genotipo de resistencia
-insitir en apego optimo
-derivar especialista

How well did you know this?
1
Not at all
2
3
4
5
Perfectly
127
Q

Diagnostico VHC

A

ELISA anti-VHC

  • Prueba confirmatoria. PCR-ARN-VHC

La PCR detecta el virus desde la 1-2da semana de la infección.
Los anticuerpos pueden detectar hasta 7-8 semanas despues de la infección.

Pacientes con ELISA (+) pero PCR (-) = Infección aguda sin desarrollar infección crónica o se trata de un falso positivo

How well did you know this?
1
Not at all
2
3
4
5
Perfectly
128
Q

VIH
cuales antihipertensivos pueden interactuar con el TAR

A

BCC que se metabolizan con CYP3A4,
interactuan con ITRNN y los IP

How well did you know this?
1
Not at all
2
3
4
5
Perfectly
129
Q

Tratamiento en pacientes con VHC

A

Elección: Interferón pegilado + ribavirina
De 24-48 semanas dependiendo del genotipo.

Simplificado: antivirales de acción directa con actividad pangenotípica.

How well did you know this?
1
Not at all
2
3
4
5
Perfectly
130
Q

VIH
cuales son las metas de TA en pacientes con VIH

A

TA debajo de 140/90
farmacos mas tolerados IECA y ARA2

How well did you know this?
1
Not at all
2
3
4
5
Perfectly
131
Q

Como es medido el grado de fibrosis hepática en VHC

A

Elastografía y marcadores serológicos directos (FibroSure, Enhanced Liver Fibrosis, Fibrosis Test) e indirectos FIB-4
Mejor método: Biopsia hepatica

How well did you know this?
1
Not at all
2
3
4
5
Perfectly
132
Q

VIH
cuales son las enfermedade renales asociadas a VIH

A

-Glomerulonefritis asociada a VIH (focal y segmentaria)
- Nefropatia vascular
- nefritis tubulointersticial asociada a tenofovir
-Daño renal agudo asociado a ARV sepsis o rabdomiolisis

How well did you know this?
1
Not at all
2
3
4
5
Perfectly
133
Q

Tratamiento en pacientes con cirrosis decompensada

A
  • Elegibles con ribavirina tratados con sofosbuvir/velpatasvir o ledipasvir/sofosbuvir (genotipos 1,4, 5 o 6) ambos en combinación por 12 semanas

-No elegibles a ribavirina mismo tratamiento pero por 24 semanas

How well did you know this?
1
Not at all
2
3
4
5
Perfectly
134
Q

VIH
PrEP
quinenes son candidatos a este trata miento

A

Hombres que tienen sexo con hombres, mujeres trans, parejas serodiscordantes sin control viral
CON VIH NEGATIVO

How well did you know this?
1
Not at all
2
3
4
5
Perfectly
135
Q

Seguimiento en pacientes VHC

A
  • Sin cirrosis con PCR solo en aquellos con riesgo de reinfección por exponerse a factores de riesgo para adquirir nuevamente la infección.
  • Con cirrosis es con un USG hepático con o sin alfa-fetoproteína cada 6 meses
How well did you know this?
1
Not at all
2
3
4
5
Perfectly
136
Q

VIH
PrEP
Cuales son los ARV de eleccion

A

TDF solo o con FTC. por evento o demanda o en modalidad diaria
TAF solo o con FTC unicamente en hombres que tienen sexo con hombres y mujeres trans en modalidad diaria

How well did you know this?
1
Not at all
2
3
4
5
Perfectly
137
Q

Virus de la Hepatitis E

A

ARN monocatenario
Familia: Herpeviridae
Provoca: hepatitis aguda autolimitada

How well did you know this?
1
Not at all
2
3
4
5
Perfectly
138
Q

VIH
en pacientes PrEP cada cuanto debe de hacerse seguimiento

A

1ra cita
2da al mes y posterior cada 4
si es <25 o usa drogas se recomeinda mensual

How well did you know this?
1
Not at all
2
3
4
5
Perfectly
139
Q

Incubación VHE

A

15-60 días

How well did you know this?
1
Not at all
2
3
4
5
Perfectly
140
Q

VIH
PrEP
cuando y como se recomienda la suspencion

A

Recomendad el tiempo que dure el periodo de riesgo, se recomienda descontinuar 28 dias despues de la ultima exposicion de riesgo

How well did you know this?
1
Not at all
2
3
4
5
Perfectly
141
Q

Causa muy común de hepatitis, pero poco diagnosticada

A

VHE

How well did you know this?
1
Not at all
2
3
4
5
Perfectly
142
Q

VIH
PEP
en quienes esta recomendada

A

personas con riesgo sustancial (transfusion, jeringas no esteriles, pinchazo percutaneo, coito anal receptivo, penetracion vaginal receptivo, coito anal insertivo, coito vaginal insertivo, sexo oral, etc)
iniciar en las primeras 72 horas

How well did you know this?
1
Not at all
2
3
4
5
Perfectly
143
Q

VIH
profilaxix para mycobacterium avium, cuales son las indicaciones

A

indicado en CD4 <50, no se indica si el paciente iniciara TAR inmediato
AZITRO-CLARITRO
alternativo: rifabutina
Suspender: con CD4>100 por >=3meses, reiniciar CD4<50

How well did you know this?
1
Not at all
2
3
4
5
Perfectly
144
Q

VIH
profilaxis para malari

A

indicados a zonas endémicas:
-doxiciclina 2 días antes de viajar, durante toda la estancia y 4 dias despues de abandonarla, mefloquina, primaquina
ALTERNATIVO: cloroquina

How well did you know this?
1
Not at all
2
3
4
5
Perfectly
145
Q

VIH
profilaxis para Pneumocystis jirovecii

A

indicado en CD4<200, aftas orales recurrentes, SIDA
-TMP/SMX
ALTERNATIVO: dapsona, dapsona+pririmetamina+acido folinico

How well did you know this?
1
Not at all
2
3
4
5
Perfectly
146
Q

VIH
profilaxis para toxoplasmosis

A

indicado en IgG positivo con CD4<100
-TMP/SMX
ALTERNATIVO: dapsona, dapsona+pirimetamina+acidofolinico

How well did you know this?
1
Not at all
2
3
4
5
Perfectly
147
Q

VIH
Profilaxis para tuberculosis

A

indicado en PPD>5mm, IGRA+, sin evidencia de TB activa, sin antecedente de tto para TB activa o latente, PPDnegativa con contacto cercano en TB pulmonar
-Isoniazida+piridoxina
ANLTERNATIVA: rifapentina+isoniazida

How well did you know this?
1
Not at all
2
3
4
5
Perfectly
148
Q

VIH
profilaxis de varicela zoster

A

indicado pre exposicion: CD4>200 no vacunados, sin infeccion previa seronegativas VVZ: vaviuna varivax 2dosis cada 3 meses, si vacuna produce enfermedad se da aciclovir
POST-exposicion: inmunoglobulina contra VVZ, administrada en 96 horas, alternativa post: aciclovir o valaciclovir.

How well did you know this?
1
Not at all
2
3
4
5
Perfectly
149
Q

VIH
profilaxis para coccidioides immitis

A

indicado en serologia positiva con CD4 <250 con enfermedad clinica descartada
-Fluconazol o itraconazol

How well did you know this?
1
Not at all
2
3
4
5
Perfectly
150
Q

VIH
profilaxis para histoplasma capsulatum

A

Indicado en riesgo ocipacional o vivir en area hiperendemica: no en uso rutinario primario
-Itraconazol

How well did you know this?
1
Not at all
2
3
4
5
Perfectly
151
Q

VIH
cual es el tratamiento indicado en candidiasis mucocutanea en personas con VIH

A

orofaringea: fluconazol 100mg c24hrs por 7-14d
esofagica: fluco 100-400 mg vo-iv c24hrs por 14-21dias
vulvovaginal no complicada: fluco 150 vo dosis unica, o azoles topicos
vulvovaginal severa o recurrente:fluco 100-200 VO casa 24 por al menos 7 dias

How well did you know this?
1
Not at all
2
3
4
5
Perfectly
152
Q

VIH
cual es el tratamiento para CMV en personas con VIH

A

retinitis: ganciclovir 2mg intravietreo por 14-21 dias
lesiones perifericas: valganciclovir 900mg vo c12 hrs
mantenimiento 900mg c24hrs por 3-6 meses
esofagitis o colitis: ganciclovir IV o valganciclovir 900 vo por 21-42 dias

How well did you know this?
1
Not at all
2
3
4
5
Perfectly
153
Q

VIH
cual es el tratamiento para criptococosis en personas con VIH

A

MENINGEA:
induccion: anfotericina B IV + fluocitosina 2 sem
consolidasion: fluconazol 8 sem
mantenimiento: fluco 12 meses
EXTRAPULMONAR no meningea severa o pulmonar: igual que la meningea
EXTRAPULMONAR no meningea leve o moderada
-fluco 400 c24 por 12 meses

How well did you know this?
1
Not at all
2
3
4
5
Perfectly
154
Q

VIH
cual es el tratamiento para histoplasmosis en pacientes con VIH

A

diseminada moderada-grave: anfo b 2 semanas
mantenimiento: itraconazol 12 meses
diseminada leve: itraconazol 12 meses
meningitis: inicio 4-6 sem anfo b, mantenimiento itra por 12 meses

How well did you know this?
1
Not at all
2
3
4
5
Perfectly
155
Q

VIH
en pacientes con VIH cual es el tratamiento para infeccion de Micobacterium avium

A

aguda: 12 meses, claritro+etambuitol o azitro+etambutol

How well did you know this?
1
Not at all
2
3
4
5
Perfectly
156
Q

VIH
en pacientes con VIH cuales es el tratameinto para infeccion por Pneumocystis jirovecii

A

moderada o severa: TMP/SMX por 21 dias por 21 dias

How well did you know this?
1
Not at all
2
3
4
5
Perfectly
157
Q

VIH
cual es el tratamiento en apcientes con VIH e infeccion por sifilis

A

primaria, secundaria o latente temprana: peni benzatinica 2.4 millones IM du
latente tardia: >1 año de duracion. penicilina benzatinica 2.4 im cada semana por tres semanas
NEUROSIFILIS
- peni g cristalina 18-24 millones IV cada 24hrs por 3 semanas

How well did you know this?
1
Not at all
2
3
4
5
Perfectly
158
Q

VIH
en pacientes con VIH y toxoplasmosis cerebral

A

aguda: pirimetamina 200 vo du, posterior 75mg y se agrega sulfazadiazina y acido folinico
mantenimiento: pirimetamina + sulfazidina + acido folinico

How well did you know this?
1
Not at all
2
3
4
5
Perfectly
159
Q

Genotipos más comunes de VHE

A

4 genotipos
y 2 prevalencen en México

How well did you know this?
1
Not at all
2
3
4
5
Perfectly
160
Q

VIH
PEP
cual es el ciclo recomendado

A

en pacientes negativos previos, que buscan atencion en las primeras 72 hrs
- 28 dias dependiendo la edad
2-12a: TDF/FTC +DRV/r
>=13a con TFG >30: BIC/TAF/FTC
>=13a con TFG<30: DRV/c+DTG
EMBARAZO:
TDF/FTC+RAL o DTG

How well did you know this?
1
Not at all
2
3
4
5
Perfectly
161
Q

Complicación de la Varicela Zóster

A

Neuralgia posthérpetica causando un dolor intenso y afecta la calidad de vida

How well did you know this?
1
Not at all
2
3
4
5
Perfectly
162
Q

Transmisión de VHE

A

Consumo de agua o comida contaminada, transfusiones, madre-hijo.

How well did you know this?
1
Not at all
2
3
4
5
Perfectly
163
Q

Tratamiento antiviral de Varicela Zóster

A

Iniciarse <72 horas de haber iniciado los síntomas

Administración sistémica de antivirales reduce los síntomas cutáneos y gravedad del dolor

-Aciclovir 800 mg 5 veces al día por 7 días
-Valaciclovir 1000 mg cada 8 horas por 7 días
-Famciclovir 1000 mg cada 8 horas por 7 días

How well did you know this?
1
Not at all
2
3
4
5
Perfectly
164
Q

Cuadro clínico VHE

A

Malestar general, anorexia, náuseas, vómito, dolor abdominal, fiebre y hepatomegalia

How well did you know this?
1
Not at all
2
3
4
5
Perfectly
165
Q

Pacientes con Herpes Zoster con implicación del nervio facial como se le conce y que se sugiere

A

Síndrome de Ramsay Hunt
Tratamiento con aciclovir iv y corticoides sistemicos (primeros 72 horas mejor resultado)

How well did you know this?
1
Not at all
2
3
4
5
Perfectly
166
Q

Manifestaciones bioquímicas para VH

A

Elevación bilirrubinas y transaminasas. Resolución bioquímica ocurre 1-6 semanas del inicio de la enfermedad.

How well did you know this?
1
Not at all
2
3
4
5
Perfectly
167
Q

Indicación para vacunación contra el virus “Varicela Zoster”

A

Prevención de HZ en mayores de 50 años

How well did you know this?
1
Not at all
2
3
4
5
Perfectly
168
Q

Complicaciones en VHE

A

Hepatitis fulminante, hepatitis colestásica o hepatitis E crónica

How well did you know this?
1
Not at all
2
3
4
5
Perfectly
169
Q

E

A
How well did you know this?
1
Not at all
2
3
4
5
Perfectly
170
Q

Mortalidad en mujeres embarazadas con VHE

A

15-25%

How well did you know this?
1
Not at all
2
3
4
5
Perfectly
171
Q

Diagnostico para VHE

A

Inicial: anticuerpos anti-VHE IgM.
Confirmación: estudio alterno de anticuerpos anti-VHE IgM, elevación de los títulos de anti-VHE IgG (elevación >5 veces al basal en 2 semanas)
Detección de ARN (PCR) en suero o heces.

Estudio inicial negativo y la sospecha es alta, se confirma con PCR

How well did you know this?
1
Not at all
2
3
4
5
Perfectly
172
Q

Tratamiento VHE

A

Soporte
Desarrollo de falla hepática fulminante hay que realizar trasplante hepatico.

Hepatitis E crónica –> Ribavirina por 12 semanas

How well did you know this?
1
Not at all
2
3
4
5
Perfectly
173
Q

Prevención para VHE

A

Evitar el consumo de agua no potable, alimentos preparados con poca higiene, mariscos, carnes o vegetales crudos

How well did you know this?
1
Not at all
2
3
4
5
Perfectly
174
Q

Virus de la Hepatitis B

A

ADN
Famialia: Hepadnavirirdae

How well did you know this?
1
Not at all
2
3
4
5
Perfectly
175
Q

Genotipos de VHB

A

9 genotipos de la A a la I
Genotipo “H” predonima en México

How well did you know this?
1
Not at all
2
3
4
5
Perfectly
176
Q

Periodo de incubación VHB

A

30-180 días

How well did you know this?
1
Not at all
2
3
4
5
Perfectly
177
Q

Método de transmisión VHB

A

Vertical 90%, percutánea, contacto sexual, contacto persona-persona con heridas en piel, tejidos infectados

How well did you know this?
1
Not at all
2
3
4
5
Perfectly
178
Q

Factores de riesgo para cirrosis en pacientes con VHB

A

Edad avanzada, carga viral alta, consumo de alcohol, coinfección con VHC, VHD y VIH, HBeAg positivo.

How well did you know this?
1
Not at all
2
3
4
5
Perfectly
179
Q

Factores de riesgo para hepatocarcinoma VHB

A

Hombres, antecedente familiar, edad avanzada, HBeAg positivo, reversión de seroconversión de HBeAg, cirrosis, genotipo C, carga viral alta, coinfección con VHC

How well did you know this?
1
Not at all
2
3
4
5
Perfectly
180
Q

Manifestaciones clínicas VHB fase aguda

A

Subclínicas 70%,
Hepatitis
Ictérica 30%
Fulminante 0-10.5%

How well did you know this?
1
Not at all
2
3
4
5
Perfectly
181
Q

Manifestaciones clínicas en la fase crónica VHB

A

Asintomática, presentarse como cirrosis o hepatocarcinoma
Manifestaciones extrahepáticas: poliarteritis nodosa, nefropatía membranosa o glomerulonefritis membranoproliferativa, anemia aplásica.

How well did you know this?
1
Not at all
2
3
4
5
Perfectly
182
Q

Diagnostico para VHE

A

Solicitar prueba incial HBsAg, anti-HBs, anti-HBc

How well did you know this?
1
Not at all
2
3
4
5
Perfectly
183
Q

Interpretación de serologías
HBsAg(-)
Anti-HBc(-)
Anti-HBs(-)

A

Susceptible

How well did you know this?
1
Not at all
2
3
4
5
Perfectly
184
Q

Interpretación serologíca
HbsAg(-)
Anti-HBc(+)
Anti-HBs(+)

A

Infección previa

How well did you know this?
1
Not at all
2
3
4
5
Perfectly
185
Q

Interpretación serologica
HBsAg (-)
Anti-HBc(-)
Anti-Hbs(+)

A

Vacunado

How well did you know this?
1
Not at all
2
3
4
5
Perfectly
186
Q

E. histolitica
Menciona su caracteristica microbiologica

A

Protista unicelular con trofozoito (patogeno activo) quiste (resistente infesante)
tipo amebiano
distribucion mundial con preferencia en trópicos, sanidad deficiente y agua contaminada

How well did you know this?
1
Not at all
2
3
4
5
Perfectly
187
Q

E. hitolitica
cual es el mecanismo de contagio

A

Ingestion por agua y alimentos contaminados, moscas y cucarachas como vectores
Practicas sexuales ano-orales

How well did you know this?
1
Not at all
2
3
4
5
Perfectly
188
Q

E. histolitica
menciona el ciclo de infeccion posterior a la ingesta

A

Quistes al contacto con acido gastrico liberan trofozoitos al duodeno, liberan lectina que se une al epitelio intestinal induciendo apoptosis, fagocita celulas muertas, fagocita proteazas y secuancialmente inicia con la formacion de ULCERAS EN MATRAZ

How well did you know this?
1
Not at all
2
3
4
5
Perfectly
189
Q

E histolitica
cual es la forma más común de amebiasis

A

Diarrea amebiana: diarrea 3 dias de media en un paciente infestado con E.hystolitica, puede manifestarse también con disenteria

How well did you know this?
1
Not at all
2
3
4
5
Perfectly
190
Q

E. hystolitica
cuales son las manifestaciones de abseso hepatico

A

VARONES 10:1MUJERES
-Fiebre, dolor hipocondrio, leucositosis, anomalidad transaminasas
-presencia de abseso en USG (multiples absesos que confluyen en uno)
abseso “frio”, se identifica pastas de anchoas o salsa de chocolate

How well did you know this?
1
Not at all
2
3
4
5
Perfectly
191
Q

E.hystolitica
cual es la mejor forma de diagnostico

A

Combinacion de serologia para E.hysto + identificacion del parasito en heces
- >sensibilidad: antigenos especificos, PCR, serologia

How well did you know this?
1
Not at all
2
3
4
5
Perfectly
192
Q

E. hystolitica
cual es el tratamiento de eleccion

A

PRIMERO
1. Metronidazol por 5-10 dias
2. Tinidazol por 5 dias
POSTERIOR
1. Paromomicina 5-10 dias
2. yodoquinolol o doloxanida por 10 dias

(Metronidazol seguido por diyodohidroxiquinoleína)

How well did you know this?
1
Not at all
2
3
4
5
Perfectly
193
Q

E. hystolitica
cuales son las indicaciones de drenaje para absceso

A

-Diametro >10cm
- falta de respuesta al tto en 72hrs
- riesgo de ruptura
- lobulo hepatico izquierdo
- gestantes
- contraindicacion para metronidazol
- complicacion pleuropulmonar

How well did you know this?
1
Not at all
2
3
4
5
Perfectly
194
Q

TRICONOMA VAGINALIS
menciona su caracteristica microbiologica

A

protozoario urogenital solo como trofozoito, 4 flagelos y membrana ondulante, division longitudinal
se encuentra en uretra vagina y prostata
distribucion mundial, transmite por contacto sexual

How well did you know this?
1
Not at all
2
3
4
5
Perfectly
195
Q

TRICONOMA VAGINALIS
cual es su periodo de incubacion

A

5-28 dias
hasta el 50% de mujeres y la mayoria de varones permanecen asintomaticos

How well did you know this?
1
Not at all
2
3
4
5
Perfectly
196
Q

TRICONOMA VAGINALIS
cual es la caracteristica de infeccion sintomatica

A
  1. descarga fetida amarillo verdosa, con laucocitos PNM, eritema vulvovaginal y ectocervical “colpitis en fresa”, PH >4.5 y prueba KOH positiva
    - se puede acompañar de dispareunia, dolor abdominal y disuria y agravarse en la menstruacion
How well did you know this?
1
Not at all
2
3
4
5
Perfectly
197
Q

TRICONOMA VAGINALIS
cuales son las opciones diagnosticas

A
  1. Microscopia de secresiones vaginales en fresco o con inmunofluorescencia directa
  2. Cultivo (sensible, tarda de 3-7 dias)
    -cito cervical con tincion PAP o Giemsa
How well did you know this?
1
Not at all
2
3
4
5
Perfectly
198
Q

TRICONOMA VAGINALIS
cuales son los agentes terapeuticos de eleccion

A
  1. Metronidazol por 7 dias O EN du
  2. Tinidazol 2da linea
    NO DAR CLINDAMICINA
    dar tto a pareja previene recurrencia
How well did you know this?
1
Not at all
2
3
4
5
Perfectly
199
Q

Interpretación serologica
HBsAg ( +)
Anti-HBc IgM e IgG (+)
Anti-HBs(-)

A

Infección aguda

How well did you know this?
1
Not at all
2
3
4
5
Perfectly
200
Q

Interpretación serologica
HBsAg (+)
Anti-HBc IgM (-)
Anti-HBc IgG (+)
Anti-Hbs(-)

A

Infección crónica

How well did you know this?
1
Not at all
2
3
4
5
Perfectly
201
Q

Interpretación serologica
HBsAg(-)
Anti-HBc(+)
Anti-HBs(-)

A

Recuperación de infección aguda, infección previa sin producción de anti-HBs, falso positivo, infección crónica con niveles indetectables de HBsAg

How well did you know this?
1
Not at all
2
3
4
5
Perfectly
202
Q

TRYPANOSOMA CRUZI
menciona su caracteristica microbiologica

A

protozoario, responsable de tripanosomiasis americana o “chagas”
vector chince besucona/triatomida/reduvida

How well did you know this?
1
Not at all
2
3
4
5
Perfectly
203
Q

TRYPANOSOMA CRUZI
como se produce la infeccion a prtir del vector

A

la chince muerde y defeca en la herida
los tripomastigotes entran por herida o al tener contacto con mucosa oral,nasal,conjuntica
la invasion es facilitada por el rascado

How well did you know this?
1
Not at all
2
3
4
5
Perfectly
204
Q

TRYPANOSOMA CRUZI
cuales son los organos a los que migra posterior a la infeccion

A

CORAZON
HIGADO
CEREBRO
ahi pierden flagelo y se convierte en amastigotes

How well did you know this?
1
Not at all
2
3
4
5
Perfectly
205
Q

TRYPANOSOMA CRUZI
cual es la presencia/clinica de enfermedad de chagas aguda

A

enfermedad en niños, puede aparecer chagoma, linfadenopatia local, signo de romaña si se inoculo en conjuntivas
sintomas: fiebre, malestar, anorexia, edema facial y de extremidades inferiores, linfadenopatia, hepatoesplenomegalia, InsuCardicongestiva

How well did you know this?
1
Not at all
2
3
4
5
Perfectly
206
Q

Tratamiento VHB

A

Aguda:
Medidas de soporte
Candidatos a tratamiento aquellos con INR >1.5 curso prolongado

Usar Entecavir, tenofovir, lamivudina, adefovir o telvibudina

Puede suspenderse después de 2 pruebas de HBsAg negativas con 4 semanas de separación.

How well did you know this?
1
Not at all
2
3
4
5
Perfectly
207
Q

TRYPANOSOMA CRUZI
en cuanto tiempo resuelve la enfermedad aguda sin tratamiento

A

6-8 semanas conforme el paciente entra a fase indeterminada

How well did you know this?
1
Not at all
2
3
4
5
Perfectly
208
Q

TRYPANOSOMA CRUZI
cuales son las manifestaciones de cardiopatia cronica por chagas

A

se presenta del 10-30% en individuos cronicamente infestados
Trastornos del ritmo, ICCongestiva, tromboembolismo
ES FR para infarto cerebral

How well did you know this?
1
Not at all
2
3
4
5
Perfectly
209
Q

TRYPANOSOMA CRUZI
cuales son las manifestaciones de enfermedad GI cronica por chagas (megaenfermedad)

A

esofago: tos, disfagia, odinofagia, regurgitacion, hipersalivacion e hipertrofia de glandulas salivales, perdida ponderal, caquexia
colon: dolor abdominal intermitente y estreñimiento cronico

How well did you know this?
1
Not at all
2
3
4
5
Perfectly
210
Q

TRYPANOSOMA CRUZI
cual es el metodo diagnostico para enfermedad aguda de chagas

A

identificacion del parasito en extendido de sangre periferica con tincion giemsa
tambien se puede usar serologia, biopsia y PCR
NOM
confirmacion por strout, cultivo o xenodiagnostico o elisa

How well did you know this?
1
Not at all
2
3
4
5
Perfectly
211
Q

Tratamiento con PegINF-alfa VHB

A

Opción en pacientes inmunocompetentes sin cirrosis, que quieren un tratamiento finito, tienen genotipo A.

How well did you know this?
1
Not at all
2
3
4
5
Perfectly
212
Q

Tratamiento en embarazadas con VHB

A

TDF

How well did you know this?
1
Not at all
2
3
4
5
Perfectly
213
Q

TRYPANOSOMA CRUZI
cual es el tratamiento para enfermedad de chagas

A

nifurtimox o benznidazol para aguda o indeterminada
CRONICA: sintomatologica

How well did you know this?
1
Not at all
2
3
4
5
Perfectly
214
Q

Pacientes con tasa de FGe <60 y VHB

A

TAF o ETV .

How well did you know this?
1
Not at all
2
3
4
5
Perfectly
215
Q

ENTEROBIUS VERMICULARIS
menciona su caracteristica biologica

A

gusano pequeño blanco (oxiuro/alfilerillo), infestacion inicia por ingesta de huevos embrionados, en el Int delgado liberan larvas que maduran de 2-6 semanas, la hembra viaja por el intestino y deposita los huevos en pliegues perianales

How well did you know this?
1
Not at all
2
3
4
5
Perfectly
216
Q

ENTEROBIUS VERMICULARIS
cual es el mecanismo de transmision

A

mano-boca, prendas y juguetes, inhalación y deglución de los huevos

How well did you know this?
1
Not at all
2
3
4
5
Perfectly
217
Q

Pacientes con tasa de FGe <15 VHB

A

ETV

How well did you know this?
1
Not at all
2
3
4
5
Perfectly
218
Q

Paciente con hemodialisis tratamiento VHB

A

ETV, TAF y TDF

219
Q

Que medicamento esta contraindicado en VHB y cirrosis

A

PegINF-alfa

220
Q

HBsAg

A

Antígeno que aparece antes de los síntomas, indica enfermedad activa y si persiste >6 meses VHB crónica

221
Q

Anti-HBs

A

Indica desaparición de enfermedad aguda e inmunidad (S de salvado, único que eleva la vacuna)

222
Q

Anti-HBc IgM

A

Primer anticuerpo en aparecer indica infección aguda

223
Q

Anti-HBc IgG

A

Indica infección previa o crónica

224
Q

HBeAG

A

Indicador de transmisión y replicación viral

225
Q

Anti-HBe

A

Indica disminución de transmisión y replicación

226
Q

ADN en VHB

A

Presencia de VHB en suero indica replicación activa

227
Q

Virus Hepatitis D

A

Virión ARN, requiere la presencia de VHB

228
Q

Factores de riesgo para VHD

A

Usuario de drogas IV, antecedente de múltiples transfusiones, hombres que tienen sexo con hombres, emigrantes de países con alta prevalencia

229
Q

ENTEROBIUS VERMICULARIS
cuales son las manifestaciones clinicas

A

prurito intenso, insomnio y cnasancio

230
Q

ENTEROBIUS VERMICULARSI
como se establece el diagnostico

A

observacion de huevos en el microscopio tras recogerlos con torunda anal (puede ser necesario recogerlos hasta 3 dais)

231
Q

ENTEROBIUS VERMICULARIS
cual es el tratamiento de eleccion

A

mebendazol, alternativo albendazol o pamoato de pirantel

232
Q

ASCARIS LUMBRICOIDES
menciona sus caracteristicas biologicas

A

gusano largo de 20-35 cm, huevo libera larva que atraviesa pared duodenal y entra a torrente porta-hepatico hasta llegar a corazon y pulmon, ñarvas libres crecen en alveolos, se expulsan en la tos y son ingeridas para volver al ID
lus huevos pueden durar hasta 75 dias despues de el inicio de la infestacion y duran hasta 2 semanas en suelo

233
Q

ASCARIS LUMBRICOIDES
menciona caracteristicas de su infeccion

A

ASINTOMATICA si la infestacion es epqueña
-puede haber daño tisular en el higado o hasta perforacion y generar peritonitis
-maraña de adultos: obstruccion, perforacion, oclusion de apendice
-fiebre, distencion abdominal, vomitosSC

234
Q

ASCARIS LUMBRICOIDES
menciona la manera de diagnosticarlos

A

EXAMEN EN SEDIMENTO EN HECES: observación de huevo fecundado
LARVAS Y EOSINOFILOS EN ESPUTO: en fase pulmonar

235
Q

Transmisión de infección VHD

A

Coinfección o sobre infección VHB

236
Q

Diagnostico VHD

A

Es necesario presencia de VHB para infectar

237
Q

Incubación VHD

A

1 a 4 meses

238
Q

Diagnostico VHD

A

IgM e IgG ARN en suero y HDAg en hígado

239
Q

Tratamiento VHD

A

Tratar hepatitis B

240
Q

Prónostico VHD

A

Cronicidad de 5% en coinfección

241
Q

NEISSERIA GONORRHOEAE
Menciona las características del agente

A
  • Coco gram(-) aerobio inmovil y no esporulador, dispuesto en parejas, con apariencia de grano de café
  • Posee pili que participan en la unión a las células y la transferencia de ADN
  • Su principal proteína de superficie es PorB, permite la entrada de nutrientes y la salida de desechos
242
Q

¿Cuál es la incidencia máxima de N. Gonorrhoeae?

A

15-24 años.
Mujeres 50% de probabilidad de adquirir la infección después de un contacto; hombres, 20%

243
Q

¿Qué requiere N. Gonorrhoeae para crecer en cultivo?

A

Temp 35-37° y suplementación de CO2, + hierro

244
Q

Diagnóstico de N. gonorrhoeae

A
  • Tinción gram de muestras uretrales es precisa sólo en hombres
  • Cultivo Agar Thayer-Martin ha sido sustituido por las pueblas de amplificación de ácidos nuclécicos → PCR, TMA, SDA
245
Q

¿Qué enfermedades produce N. Gonorrhoeae?

A
  • Gonorrea (sexual)
  • Gonococcemia → infecciones diseminadas
  • Oftalmia gonocócica
  • Perihepatitis (Sx Fitz-Huhg-Curtis)
  • Gonorrea anorrectal
  • Faringitis
246
Q

Tratamiento de primera y segunda línea para gonorrea

A
  1. Ceftriaxona 500 mg IM DU + azitromicina 1gr VO DU
  2. Cefixima 400 mg VO DU + azitromicina 1gr VO DU
247
Q

¿Cuáles son las características clínicas de gonorrea?

A

Secreción purulenta en la localización afectada, tras periodo de incubación de 2-5 días
- Hombres 95% sintomáticos → descarga uretral y disuria
- Mujer puede ser asintomática o con descarga vaginal, disuria, dolor abdominal, metrorragia, sangrado poscoital y friabilidad cervical. 10-20% presenta complicaciones → salpingitis, abscesos tuboováricos, infección de las glándulas de Bartholin y Skene y EPI.

248
Q

Tratamiento para enfermedades causadas por N. Gonorrhoeae: faringitis, conjuntivitis, embarazo y lactancia e infección diseminada

A
  • Faringitis: ceftriaxona 500 mg IM DU + azitromicina 1gr VO DU
  • Conjuntivitis: ceftriaxona 500 mg IM x 3 días
  • Embarazo y lactancia: ceftriaxona 500 mg IM DU + azitromicina 1 gr VO DU o espectinomicina 2 gr IM + azitromicia 1 gr VO DU
  • Diseminada: ceftriaxona 1 gr IM o IV c/24 hrs hasta cumplir 24-48 hrs de memoria clínica; continuar con cefixima 400 mg VO c/12 hrs x 7 días
249
Q

Inhibidores de transcriptasa reversa análogo de nucleósidos

A

Abacavir (ABC)
Emtricitabina (FTC)
Lamivudina (3TC)
Zidovudina (AZT-ZDV)

250
Q

Inhibidores de transcriptasa reversa analogos de nucleotidos

A

Tenofovir
-Disoproxil TDx .- causa ER y osteoporosis
-Alafenamida TAF.- más seguros

251
Q

Inhibidores de transcriptasa reversa no nucleosidos

A

-Doravirina (DOR) mejor y más seguro
-Efavirenz (EFV).- efectos secundarios terrores nocturnos, teratogenico (defectos del tubo neural)
-Nevirapina (NVP)
-Rilpivirina (RPV)

252
Q

Inhibidor de proteasa

A

-Atazanavir (ATV)
-Darunavir (DRV)
-Indinavir (IDV)
-Lopinavir (LPV)
-Ritonavir (RTV)

253
Q

Inhibidor integrasa

A

-Bictegravir
-Dolutegravir
-Raltegravir
-Elvitegravir

254
Q

Primera y segunda línea tratamiento VIH

A

1° Tenefovir + Emtricitabina + Bictegravir
2° Abacavir + lamivudina + Dolutegravir

255
Q

Bacilo Gram(+) móvil, con esporas terminales, anaerobio estricto, con actividad proteolitica

A

Clostridium Tetani

256
Q

Factores de virulencia de Clostridium Tetani

A

-Tetanoespasmina y Tetanolisina

257
Q

Que es la tetanoespasmina

A

Neurotoxina termolábil que bloquea la liberación de GABA en las sinapsis al unirse irreversiblemente a la endopeptidasa de cinc después de llegar al soma de la neurona motora en la médula espinal.

258
Q

Que es la tetanolisina

A

Hemolisina termolabil inhibe con el oxígeno y el colesterol sérico

259
Q

Tiempo de incubación de clostridium tetani

A

Duración directamente proporcional a la distancia entre la herida primaria y el SNC 3-21 días, media de 8 días.

260
Q

Diagnostico Clostridium tentani

A

Clinica

261
Q

Como se presenta Tétanos generalizado

A

Más común
Espasmos musculares generalizados (trismus, opistótonos) afectación del SNA (sialorrea, arrtimias cardiacas, fluctuaciones de la TA, diaforesis profusa, deshidratación, rabdomiolisis, espasmo laríngeo, retención urinaria)

262
Q

Como se presenta Tétanaos localizado

A

Espasmos musculares a una área localizada, infección primaria
Pronostico bueno

Tétanos cefalico: infección del oído medio –> disfunción de los nervios craneales VII y puede evolucionar a tétanos generalizado

263
Q

Como se presenta tétanos neonatal

A

Infección neontal afecta principalmente al muñón umbilical. Mortalidad elevada >90%

264
Q

Tratamiento en Clostridium tetani

A

Soporte ventilatorio + control de espasmos (BZD) metronidazol + inmunoglobulina tetánica humana 500 UI IM

265
Q

Bacilo espiral de extremos afilados, activamente móvil y anaerobio estricto, muy sensible a los agentes fisicos y quimicos con propiedades antibacterianas

A

Treponema Pallidum

266
Q

Periodo de incubación de treponema Pallidum

A

21 días

267
Q

Sifilis primaria

A

Microscopia de campo oscuro, inmunofluorescencia o identificación del agente en biopsia.
Cháncro (úlcera cútanea pequeña, indolora) adenopatias regionales

Duración 2-6 semanas

268
Q

Sifilis secundaria

A

Hallazgos a la exploración y reactivos seológicos (VDRL y reagina plasmpatica rápida) confirmación con absorción de anticuerpos treponémicos fluorescentes o identificación del agente en biopsias o fluidos corporales

Exantema maculopapular simétrico, lesiones palmo-plantares, condilomas planos y alopecia areata

Duración 7 a 8 semanas

269
Q

Sifilis Latente

A

Hallazgos a la exploración y reactivos seológicos (VDRL y reagina plasmpatica rápida) confirmación con absorción de anticuerpos treponémicos fluorescentes o identificación del agente en biopsias o fluidos corporales

Asintomatica, serología positiva
<1 año temprana y >1 año tardía

270
Q

Sifilis Terciaria

A

Sifilis cardiovascular (ins. aórtica) o neurosilifis (tabes dorsal) y gomas (lesiones granulomatosas
Inicio: 25 años despues

271
Q

Sifilis venérea primaria incubación y clinica

A

21 días incubación

Chancro duro (pene,vagina,ano, boca) NO dolorosa, fondo limpio, sin exudado y normalmente único.
Adenopatias inguinales, bilaterales, no dolorosas no supuran y duras.

272
Q

Sifilis venérea secundaria clinica

A

7-8 semanas despues de la aparición del chancro .
Condilomas planos, papúlas planas hipertróficas de color rosado mate que se manifiestan en las uniones mucocutáneas

273
Q

Sifilis venérea terciaria clinica

A

> 20% de los pacientes no tratados hasta 25 años después de la infección
-Sifilis tardía benigna (gomas)
Aparece 1-10 años después de la infección. Lesiones granulomatosas destructivas asimétricas, agrupadas con dolor nocturno. en hueso, piel y otros tejidos.

-Sifilis cardiovascular
Aparecer 5-10 años despues de la infección, debe a endasteritis de los vasa vasorum. Princiaples complicaciones: insuficiencia aórtica y aneurisma aórtico

274
Q

Neurosifilis

A

Meningitis sifilítica: parálisis de nervios craneales unilaterales o bilaterales

Sifilis meningovascular: Trombosis cerebrovascular e infarto

Tabes dorsalis: Pérdida de reflejos periféricos y propiocepción, ataxia, incontinencia urinaria

Paresia general: Inicia con manifestaciones demenciales

275
Q

Diagnostico de primera elección sifilis

A

Visualización de organismos delgados en forma de sacacorchos en microscopio de campo oscuro

276
Q

Pruebas treponemicas (FTA-ABS)

A

Permiten la confirmación del diagnostico

277
Q

Pruebas no treponemicas (VDRL)

A

Son muy sensibles pero poco especificas, tamizaje

VDRL con niveles >1:8

278
Q

Tratamiento en sífilis temprana (primaria, secundaria y latente temprana)

A

Penicilina G benzatinica 2.4 millones de UI IM en DU

-Alérgicos a penicilina: Desensibilizar (incluyendo el embarazo)

279
Q

Tratamiento Sifilis latente tardía (>1 año) terciaria con gomas o cardiovascular

A

Inyecciones Penicilina G benzatinica adicionales de 2,4 millones de unidades a los 7 y 14 días de inyección inicial

280
Q

Tratamiento neurosifilis

A

Penicilina G benzatinica 2,4 millones de unidades IM una vez al día más 500 mg de probenecid VO 4 veces al día por 17 días

281
Q

Cuando se considera falla al tratamiento en sifilis

A

No hay disminución de la titulación en 4 veces la basal después de 6 meses de inicial el tratamiento.

282
Q

Bacilo Gram (-) pequeño sin capa de peptidoglucanos en su pared celular. Párasito intracelular estricto del ser humano

A

Chlamydia trachomatis

283
Q

Ciclo de vida de chlamydia trachomatis

A
  • Cuerpo elemental infeccioso
    -Cuerpo reticular no infeccioso
284
Q

Biovariantes asociadas con la enfermedad humana en Chlamydia trachomatis

A

Tracoma
Linfogranuloma venéreo

285
Q

Factores que contribuyen a la virulencia en Chlamydia trachomatis

A

Replicación intracelular, capcidad de evitar la fusión del fagosoma con los lisosomas.

286
Q

Tracoma como chlamydia trachomatis

A

Proceso granulomatoso inflamatorio crónico de la superficie del ojo que provoca ulceración corneal, cicatrización, formación de pannus y ceguera

287
Q

Conjuntivitis de inclusión en los adultos por Chlamydia trachomatis

A

Proceso agudo con secreción mucopurulenta, dermatitis, infiltrados y vascularización corneales en la enfermedad crónica

288
Q

Conjuntivitis neonatal por chlamydia trachomatis

A

Proceso agudo secreción mucopurulenta con síntomas posterior a la primera y segunda semana de nacimiento

289
Q

Neumonia del lactante por Chlamydia trachomatis

A

Periodo de incubación de 2-3 días seguido de rinitis, acompañada de bronquitis con una tos no productiva caracteristica

290
Q

Infecciónes urogenitales por Chlamydia trachomatis

A

Uretritis y cervicitis
Afectación aguda de aparato genitourinario
Secreción mucopurulenta
Infecciones asintomaticas frecuentes en mujeres.

291
Q

Linfogranuloma venereo por Chlamydia trachomatis

A

Úlcera indolora desarrollada en el sitio de la infección, desaparece espontaneamente seguida de adenopatias locales y posterior aparición de sintomas sistemicos

292
Q

Diagnostico Chlamydia trachomatis

A

Cultivo es muy especifico relativamente insensible
Pruebas de antigeno (DFA, ELISA) relativamente insensible
Pruebas de amplificación molecular más sensibles

293
Q

Diagnostico de linfogranuloma venéreo

A

Serología positiva (>1:64) serotipos L1, L2 y L3

294
Q

Diganostico de uretritis no gonocócica y cervicitis mucopurulenta

A

Confrimación por cultivos, inmunofluorescencia o reacción en cadena de la ligasa (LCL)

295
Q

Esquema terapeutico en linfogranuloma venereo

A

Drenaje por aspiración de los bubones solo ante la ruptura inminente
- Doxiciclina por >=3 semanas
- Eritromicina por >= 3 semanas

296
Q

Esquema terapeutico en uretritis no gonocócica

A

Azitromicina DU (gestantes)
Doxicilina por 7 días
Eritromicina por 7 días
Ofloxacino por 7 días
Levofloxacino por 7 días

297
Q

Tratamiento para tracoma

A

elección azitromicina du
Alternativa:
- Tetraciclina ungüento oftálmico (dos aplicaciones diarias por 6 semanas)

298
Q

Como inicia la enfermedad por virus herpes simple

A

Contacto directo

299
Q

Donde establece su latencia y donde se reactiva el virus herpes simple

A

Latencia en neuronas
Reactiva por estrés o inmunosupresión

300
Q

Como se transmite VHS-1

A

Vía oral

301
Q

Como se transmite VHS-2

A

Contacto sexual

302
Q

Herpes labial

A

Lesiones en comisuras bucales o junto labios, activadas desde los ganglios trigéminos

303
Q

Faringitis herpética

A

Gingivoestomatitis herpética, úlceras en lengua, paladar y encías

304
Q

Queratitits herpética

A

Unilateral y la enfermedad recurrente, originar cicatriz permanente, lesiones corneales o ceguera

305
Q

Panadizo herpético

A

Infección en los dedos

306
Q

Herpes de los gladiadores

A

Inicia a traves de cortes o abrasiones de la piel, de lucha o rugby

307
Q

Herpes genital

A

Hombre: lesiones en el glande o cuerpo del pene
Mujer: lesiones vulvares, vaginales, cervicales, perineales en cara interna de musculo, dolorosas acomapañdas de prurito y secreciones vaginal mucoide

308
Q

Encefalitis herpética

A

VHS-1 limitarse a uno de los lóbulos temporales, convulsiones, anomalías neurológicas focales y otras.

309
Q

Infección neontal:

A

Severa mortal
VHS-2 y adquiriendose canal de parto o in útero
Presenta con septicemia y lesiones vaculares
Ausencia de inmunidad celular el virus se disemina al hígado, pulmón y SNC—> muerte, retraso mental o incapacidad neurológica

310
Q

Microscopia de las células de la base de la lesión en un frotis de Tzank

A

Células gigantes multinucleadas y cuerpos de unclusión de Cowdry tipo A

311
Q

Definición de caso sospechoso VHS

A

Paciente con lesiones compatibles u tinción Tzank positiva

312
Q

Definición de caso confirmado para VHS

A

Cultivo, inmunofluorescencia o PCR

313
Q

Primer episodio de herpes genital tratamiento

A

Aciclovir, famciclovir o valaciclovir por 7 a 10 días

314
Q

Herpes genital recurrente tratamiento

A

> 6 episodios anuales
Aciclovir, famciclovir o valaciclor por 5 días

315
Q

Herpes genital grave o complicado tratamiento

A

Aciclovir IV por 2-10 días o hasta la resolución clínica

316
Q

Coinfección por VIH y VHS tratamiento

A

Aciclovir, famciclovir o valaciclovir por 5 a 10 días

317
Q

VHS embarazo

A

Aciclovir por 10 días

318
Q

Herpes neonatal

A

Aciclovir por 21 días

319
Q

Virus Epstein Barr

A

Agente de mononuclosis infecciosa positiva, anticuerpos heterófilos.
Relacion etiologica con linfoma africano (endémico) de Burkitt, enfermedad de Hodgkin y carcinoma nasofaríngeo

320
Q

Manifestaciones clínicas mononuclosis infecciosa

A

Cefalea leve, fatiga, esplenomegalia, ocasionalmente hepatitis y exantema inducido por ampicilina

Tríada clásica:
Fiebre, linfadenoaptia y faringitis exudativa

321
Q

Diagnostico de mononuclosis infecciosa inducida por el VEB

A

Síntomas.
Linfocitos T atípicos (células de Downey
Linfocitosis, antiucepros heterófilos, anticuerpos frente a los antígenos virícos

322
Q

Tratamiento para VEB

A

Esteroides y aciclovir solo cuando se presenta mononucleosis complicada (con dificultad respiratoria por edema laríngeo o dolor abdominal intenso)

323
Q

Causa virica más frecuente de anomalias congénitas

A

CMV

324
Q

Transmisión de CMV

A

Contacto oral o sexual, transfusiones, trasplantes, in utero, canal de parto y durante lactancia

325
Q

Manifestaciones por CMV

A

Síndrome de mononucleosis o postransfusión
Complicarse con carditis o hepatitis, polineuritis y mielitis

326
Q

Manifestaciones por CMV en neonatos infectados

A

Sordera, calcificaciones intracerebrales, microcefalia y retraso mental

327
Q

Caracteristica histologica de la infección por CMV

A

Célula citomegálica, contiene un cuerpo de inclusión intranuclear basófilico central denso, “ojo de búho”.

Se observan en tinciones papanicolaou y hematoxilina-eosina

328
Q

Tratamiento CMV pacientes inmunosuprimidos

A

Ganciclovir, valganciclovir, cidofovir y foscarnet

329
Q

VHH6 (A y B) y VHH7

A

Género: Roseolavirus
Subfamilia: Betaherpesvirinae

330
Q

Como se transmite el VHH6

A

Secreciones bucales

331
Q

Que patologia causa el VHH6B o VHH7

A

Exantema súbito o róseola o sexta enfermedad

332
Q

Clinica de exantema súbito, róseola o sexta enfermedad

A

Rápida aparición de fiebre elevada, dura unos días, seguida por exantema generalizado mantiene por 24-48 horas.
Puede asociarse a crisis convulsivas febriles

333
Q

VHH8

A

Asociado al sarcoma de Kaposi (VSK)

334
Q

Origen de infección del VHH8

A

Transmisión sexual

335
Q

Virus de la Rabia

A

Genero: Lyssavirus
Familia: Rhabdoviridae
Una molécula de ARN monocatenario negativo

336
Q

Como se transmite la rabia

A

Saliva y se adquiere principalmente por mordedura de un animal rabioso.
Fuente secundaria: suspensión de virus en el aire de las cuevas, hay murciélagos rabioso

337
Q

Incubación del virus de la rabia

A

Determinada por la dosis infecciosa y por la proximidad del sitio de inoculación al SNC, infecta nervios perifericos y asciende hasta alcanzar el cerebro.

338
Q

No exposición al virus de la rabia

A

Contacto sin lesión, ningún contacto o contacto indirecto

339
Q

Exposición leve al virus de la rabia

A

Lameduras en piel erosionada o mordeduras superficiales y rasguños que incluyen dermis, epidermis y tejido celular en tronco y miembros inferiores

340
Q

Exposición grave al virus de la rabia

A

Mordeduras superficiales en cabeza y cuello, miembros superiores y genitales, mordeduras múltiples, mordeduras profundas en cualquier parte del cuerpo, agresión de un animal silvestre o cualquier animal no identificado.

341
Q

Que se debe de realizar con el animal agresor por virus de rabia

A

Observación bajo aislamiento por 10 días con un médico veterinario zootecnista

342
Q

Manejo de herida o lesión provocada por un animal en sospecha de rabia

A
  • Lavar región afectada con jabon abundane y agua a chorro durante 10-15 minutos
    Mucosa ocular: lavará por instilación profusa con solución fisiologica por 5 minutos.
  • Desinfectar la herida con agua oxigenada, alcohol al 70%, tintura de yodo o solución de yodo al 5% solución acuosa amonio cuaternario al 1%
    -Sutura debe protegerse, si es necesario suturar inmediatamente, procedera primero a aplicación del suero antirrabico hiperinmune
343
Q

Tratamiento virus de la rabia en exposición leve

A
  • Vacuna de cerebro de ratón lactante: catorce dosis diarias consecutivas de 1ml SC.
  • Vacunas de cultivos en células: cinco dias días 0,3,7, 14 y 30 IM. región deltoidea (niños pequeños cara anterolateral externa del muslo)
344
Q

Tratamiento virus rabia exposición grave

A

Aplicación de suero antirrábico heterólogo o gamma globulina antirrábica humana u vacuna antirrabica humana.
Más pronto posible
- Gamma globulina antirrábica humana 20 UI/kg DU, infiltrando la mitad de la dosis alrededor de la herida y el resto IM

  • Suero heterólogo equino 40 UI/kg previa prueba de sensibilidad.
  • Vacuna de cerebro de ratón lactante; dosis diaria durante catorce días consecutivos. tres dosis días 24,34 y 104
  • Vacunas de cultivos en células; cinco dosis IM, días 0, 3, 7, 14 y 30
345
Q

Esquema alternativo personas que acuden después de 14 días de agresión en virus de la rabia

A

Suero antirrábico heterólogo o gamma globulina antirrábica humana y dos inyecciones de vacuna de cultivos en células por vía IM en sitios separados al día 0, seguidas de una inyección en los días 7 y 21 (4 dosis)

346
Q

Prevención de la enfermedad de la rabia

A

Vacunación humana:
- Vacuna de cerebro de ratón lactante tipo fuenzalida: cuatro dosis de 1ml SC en los días 0, 7, 14 y 45 por vía subcutanea

347
Q

Vacuna para virus de la rabia de cultivos en células

A

Tres dosis IM en la región deltoidea días 0, 7 y 21 o 28 con dosis de:
- 1 ml células diploides humanas
- 0.5 ml células VERO
- 1 ml vacuna purificada de embrión de pollo

348
Q

Titulación de anticuerpos séricos en virus de la rabia

A

tres semanas después de la última dosis; si no se encuentra un nivel mínimo de 0.5 UI/l se aplicara una dosis más

349
Q

Dengue

A

Enfermedad infecciosa endémica en la mayoría de los países tropicales y subtropicales

350
Q

Agente causal del dengue

A

Virus dengue
ARN monocatenario
Familia: flaviviridae
Cuatro serotipos estrechamente relacionados (DEN-1, DEN-2, DEN3, DEN4)

351
Q

Como se transmite el dengue

A

Mosquito Aedes
“Aedes aegygpti”

352
Q

Factores de riesgo para el dengue

A

Residencia o viaje a zonas endémicas
1. edad, seoritpo involucrado, carga genética del hospedero, infección previa

353
Q

Periodo de incubación del dengue

A

3-10 días

354
Q

Fases del dengue

A

Incubación 3-10 días
Febril 2-7 días
Crítica o de fuga plasmatica entre tercer y séptimo día del inicio de la fiebre
Recuperación o reabsorción de líquidos entre séptimo y décimo día

355
Q

Dengue clásico “Fiebre rompe huesos”

A

Enfermedad febril aguda acompañada por cefalea, dolor retro-ocular, fatiga , sintomas respiratorios y gastrointestinales leves y mialgias y artralgias.

Incubación 3-14 días
Fiebre dura 5-7 días

Hallazgos clínicos erupción maculopapular “islas blancas en el Mar rojo”, linfadenopatia, hiperemia faríngea, inyección conjuntival, manifestaciones hemorrágicas (petequias y púrpura)

Infección durante el embarazo: trabajo de parto y nacimiento prematuro, muerte intrauterina, neontal y transmisión materno fetal.

356
Q

Dengue Grave

A

<3%
4-7 días después del inicio se desarrollan la fuga plasmatica y las manifestaciones hemorragicas

Fuga plasmatica hipoproteinemia, edema periférico , ascitis, hematocrito.

Trombocitopenia caracteristica con petequias, equimosis, hemorragias mucosas y sangrado gastrointestinal
Signos de alarma: dolor abdominal, vómito persistente, defervescencia abrupta, cambios en el estado mental y dificultad respiratoria

357
Q

Abordaje de dengue

A

Detección antígeno NS1 en suero (primer al quinto día, diagnostico confirmatorio)

Anticuerpos IgM (sexto al día 35°)

Anticuerpos IgG en caso de negatividad de IgM

Aislamiento del suero, LCR o biopsia hepática

Reacción en cadena de polimerasa

358
Q

Manejo del dengue no grave sin signos de alarma

A

Hidratación vida suero oral, control térmico con médios físicos y paracetamol, evitar medicamentos intramusculares, antiinflamatorios no esteroideos, esteroides, inmunoglobulinas y antivirales.

359
Q

Manejo hídrico pacientes grupo B con dengue

A

Soluciones NaCl 0.9% o Hartmann en infusiones (disminuidas secuencialmente de acuerdo con la respuesta) de 5-7 ml/kg/hora por 1-2 horas, 3-5 ml/kg/hora por 2-4 horas y <=2-3 ml/kg/hora como mantenimiento

360
Q

Manejo grupo C con dengue

A

Unidad de cuidados intensivos, se recomienda empleo de concentrados eritrocitarios casos de estado de choque.

361
Q

Recomendación de concentrados plquetarios en pacientes con dengue cuando:

A

Conteo plaquetario <5,000
Conteo plaquetario 5,000-30,000
Necesidad de realizar procedimientos quirurgicos, obstétricos o invasivos, aun cuando el recuento plquetario sea >= 50,000 plaquetas/mm3

362
Q

Cryptococcus

A

Levaduras encapsuladas esféricas u ovales, altamente neurotrópicas, diámetro de 2-20 um.

Tinción con tinta china muestra halos que representan la cápsula polisácarida extracelular
Puede detectarse fácilmente tinción de mucicarmín de Mayer.

363
Q

Cryptococcus neoformans

A

Distribución global como saprófito del suelo, en presencia de excretas de paloma.

Patógeno oportunista

364
Q

Causa más común de meningitis fúngica

A

Cryptococcus neoformans tiende a presentarse en sujetos con defectos de la inmunidad celular.

365
Q

Conteo de linfocitos CD4 para infección de C. neoformans

A

Pacientes con SIDA con conteo <200 mm3. Riesgo elevado de desarrollar criptococosis del SNC.

366
Q

Diagnostico de criptococosis meníngea

A

Detección directa del antígeno polisacarido capsular en el suero o LCR

367
Q

Características en LCR en meningitis criptococócica

A

Pleocitosis a expensas de mononucleares e hiperproteinorraquia

368
Q

Tinción para diagnostico rapido por meningitis criptococócica

A

Tinta china
Tinción plata metanamina de Grocott tambien se emplea para hongos

369
Q

Prónostico y mortalidad en pacientes con y sin SIDA con Criptococosis meníngea

A

Pronostico —> mejorado
Mortalidad 100%

370
Q

Secuelas potenciales en criptococosis meníngea

A

Demencia, hipoacusia y pérdida visual

371
Q

Profilaxis primaria criptococosis

A
  • Fluconazol 200 mg en tres dosis/semana profilaxis primaria en sujetos infectados por el VIH con negatividad para el antígeno criptocócico y conteos de linfocitos CD4 <100 cél/mm3.
372
Q

Manifestacion de infección del SNC por cryptococcus

A

Meningoencefalitis subaguda o crónica, cefalea progresiva en intensidad por varias semanas + rigidez nucal, letargo, alteraciones de la personalidad, confusión, anormalidades visuales, náusea y vómito

373
Q

Tratamiento en infección del SNC por cryptococcus

A

Inducción: anfotericina B (0.7-1 mg/kg/día)
Flucitosina (100 mg/kg/día) por >=2 semanas seguido
Consolidación: fluconazol (400 mg/día) por >=8 semanas
Supresión: fluconazol (200 mg/día) hasta alcanzar 12 meses de tratamiento antifúngico y contar con conteo de linfocitos CD4 + >100/uL y carga viral negativa

374
Q

Infección pulmonar por cryptococcus manifestaciones

A

Desde asintomatico hasta la neumonía fulminante bilateral, infiltrados pulmonares unilaterales o bilaterales

375
Q

Tratamiento en infección pulmonar por cryptococcus

A

Fluconazol (400 mg/día) por 6-12 meses

376
Q

Mecanismo de los imidazoles

A

Inhibición de la 14-alfa esterol desmetilasa, alterando la biosintesis del ergosterol para la membrana citoplasmatica

377
Q

Forma infecciosa de Giardia

A

Trofozoito
(en duodeno)

378
Q

Vía de transmisión de Giardia

A

Fecal - oral
Quistes

379
Q

Factores de riesgo para Giardiasis

A

Tener de 1-9 años
Consumo de agua contaminada (albercas, ríos y lagos)
Practicar sexo oral y anal
Antecedente de viaje a Rusia

380
Q

Clínica de Giardiasis

A

Dolor abdominal y diarrea, flatulencias, meteorismo, malabsorción, retraso en el crecimiento, esteatorrea, deficiencia de vitaminas KADE

381
Q

Diagnóstico de Giardiasis

A

Coproparasitoscópico seriado
(3 muestras con 2 días de deferencia)
Se identifican quistes y trofozoitos

382
Q

Tratamiento de Giardiasis

A

Metronidazol VO 250 mg c/ 8 horas por 5 días
Si hay resistencia: Metronidazol + Crinacrina

Tratar a portadores asintomáticos, trabajadores de guarderias, hospitales o que trabajan con alimentos

383
Q

Gols Standard de Giardiasis

A

Biopsia de duodeno

384
Q

A quién se le realiza antígeno fecal para diagnóstico de Giardiasis?

A

Pediátricos con sospecha y series negativas (CPS)

385
Q

Agente etiológico de pediculosis

A

Pediculus humanus Var. Capitis

386
Q

Clínica de pediculosis

A

Prurito constante, escoriaciones e impetiginización secundaria, adenopatías cervicales, conjuntivitis, fiebre y malestar general

387
Q

Gold Standard de pediculosis

A

Observación directa de piojos vivos, liendres o ninfas

388
Q

Sitio de localización de liendres en pediculosis

A

Región occipital y retroauricular a 1-2 mm de piel cabelluda

389
Q

Tratamiento de elección en pediculosis

A

Permetrina al 1% en crema o loción
Alt: Piretrina con piperonilo

390
Q

Tratamiento de infestaciones masivas o refractarias en pediculosis

A

Ivermectina
Alt: TMP/SMX, Albendazol, Dimeticona

391
Q

Diagnóstico diferencial de Pediculosis

A

Escabiosis, pitiriasis, dermatitis seborreica, piedra negra, piedra blanca, moldes de queratina y residuos

392
Q

Pediculosis
- Cuál es la medida de las liendres?
- Cuántos piojos pone una hembra al mes?

A
  • 0.8 mm de longitud
  • 150 huevos
393
Q

Agente etiológico de Angina de Ludwig

A

Estreptococos anaerobios

394
Q

Clínica de Angina de Ludwig
- Menciona la apariencia característica de la enfermedad

A

Síntomas de absceso dental que progresa a edema del piso de la boca, dolor, fiebre, dificultad para la deglución
- Apariencia en cuello de toro

395
Q

Estudio de imagen que evalúa la extensión de la Angina de Ludwig

A

TAC o RM

396
Q

Tratamiento de Angina de Ludwig

A

Estabilización de la vía aérea, intervención quirúrgica con remoción de piezas dentales afectadas
Ampicilina-Sulbactam o Penicilina G + Metronidazol

397
Q

Tasa de mortalidad de Angina de Ludwig

A

0-4%

398
Q

Manifestación más frecuente de toxoplamosis en población que vive con VIH

A

Encefalitis

Sobre todo en enfermedad avanzada con CD4 menores a 100

399
Q

Cuadro clínico de encefalitis por toxoplasmosis

A

Cefalea, confusión y fiebre, datos de focalización neurológica (crisis convulsivas)

400
Q

Hallazgo radiológico característico de toxoplasmosis

A

Lesión con reforzamiento en anillo

401
Q

Diferencial más importante de toxoplasmosis

A

Linfoma primario del SNC

402
Q

Tratamiento de primera línea de toxoplasmosis

A

Pirimetamina/Sulfadiazina + Ácido folínico

403
Q

Prueba diagnóstica más específica para toxoplasmosis

A

PCR en LCR 96-100%

404
Q

Complicación de toxoplasmosis

A

Secuelas neurológicas: marcha atáxica

405
Q

Agente etiológico de malaria

A

Protozoarios del género Plasmodium
Más común en México: P. vivax
Más letal: P. falciparum (no hay en México)

406
Q

Diagnóstico de malaria

A

Observación de parásitos en gota gruesa de sangre con tinción Giemsa
(3 muestras con diferencia de 12 horas)

407
Q

Vector de transmisión de malaria

A

Hembra del mosquito Anopheles

408
Q

Manifestaciones de malaria no complicada / Labs

A

Fiebre intermitente en PICOS, escalofríos, mialgias, cefalea, sudoración, debilidad, tos, diarrea, ictericia, debilidad, hepatoesplenomegalia y taquipnea
Labs: Anemia, trombocitopenia, hiperbilirrubinemia, elevación de transaminasas hepáticas

409
Q

Manifestaciones de malaria grave

A

Postración, taquipnea y alteración del estado mental
Lesión pulmonar aguda, síndrome de insuficiencia respiratoria aguda, lesión renal aguda (NTA y acidosis), malaria cerebral (lesiones pálidas en retina, cambios de coloración de los vasos y hemorragias)

410
Q

Factores protectores contra malaria

A

Anemia falciforme
Talasemias
Hemoglobina C
Deficiencia de G6PD

411
Q

Principal factor de riesgo de malaria

A

Visitar o vivir en África subsahariana

412
Q

Forma infectante de Plasmodium

A

Esporozoito

413
Q

Tratamiento de malaria

A

Cloroquina + Primaquina por 4 días, posterior Primaquina por 2 días

414
Q

Agente etiológico de osteomielitis

A

S. aureus

415
Q

Factores de riesgo de osteomioelitis

A

Traumatismo
Cuerpo extraño
Insuficiencia vascular
DM

416
Q

Qué nombre recibe la osteomielitis causada por M. tuberculosis?

A

Enfermedad de Pott
Región dorsal
Predilección por columna vertebral, considerar en paciente con osteomielitis espinal

417
Q

Gold Standard de osteomielitis

A

Cultivo del hueso infectado

418
Q
A
419
Q

Estándar de diagnóstico temprano de osteomielitis hematógena según la GPC

A

RM con gadolinio

420
Q

Tratamiento de osteomielitis

A

Dicloxacilina 100 mg/kg/día dividido en 4 dosis con aminoglucósido (Amika o Genta)

421
Q

¿Cuál es el conteo de linfocitos T CD4 en paciente con VIH que se considera como factor de riesgo para infección por P. Jirovecii?

A

<200 células/mm3

422
Q

Hallazgo clásico de P. Jirovecii en la Rx de tórax

A

Infiltrados intersticiales difusos bilaterales que se extienden a partir de la región perihiliar

423
Q

¿Cómo se confirma el diagnostico de infección por P. Jirovecii?

A

Demostrando la presencia del organismo en una muestra de esputo inducido (nebulización con solución hipertónica al 3%)

424
Q

¿Cuál es la tinción más utilizada para el diagnostico de P. Jirovecii?

A

Metenamina de plata Gocott-Gomori

425
Q

¿Método más sensible para el diagnóstico de P. Jirovecii?

A

PCR

426
Q

¿Cómo se clasifican las neumonías por P. Jirovecii utilizando el PAO2-PaO2?

A

Leve <35 mmHg, moderada 35-45 mmHg, severa >45 mmHg

427
Q

¿Cuál es la meta a alcanzar con oxígeno suplementario?
(P. Jirovecii)

A

> 60 mmHg

428
Q

Tratamiento de elección para neumonía por P. Jirovecii

A

TMP-SMX 15-20 mg/kg / 75-100 mg/kg

429
Q

Duración del tratamiento de P. Jirovecii en pacientes que viven con VIH

A

21 días

430
Q

Agente etiológico en artritis infecciosa

A

Cocos grampositivos 75%
S. aureus

431
Q

Factores de riesgo para infección de bacilos gramnegativos en artritis infecciosa

A

Enfermedades crónicas como cáncer, diabetes y cirrosis

432
Q

¿Qué bacteria es responsable de la artritis infecciosa en pacientes con varicela?

A

S. Pyogenes

433
Q

¿Cuáles son los factores predisponentes para artritis infecciosa?

A

Artritis reumatoide, articulaciones protésicas, edad avanzada e inmunosupresión

434
Q

Cuadro clínico de la artritis infecciosa

A

Instauración aguda de dolor y edema articular, limitación en el arco de movilidad, derrame articular, hipertermia circundante y fiebre.

435
Q

¿Cuáles son las articulaciones mas afectadas en artritis infecciosa?

A

Rodilla y cadera

436
Q

¿Cuál es el patrón típico en el síndrome de dermatitis-artritis gonocócico?
(Artritis infecciosa)

A

Poliartritis o tenosinovitis migratoria con pequeñas pústulas en el tronco o las extremidades

437
Q

¿Cuáles son los métodos de elección para el diagnóstico de artritis infecciosa?

A

Artrocentesis para tinción de gram, cultivo y análisis del líquido sinovial

438
Q

¿Cuál es el estándar de oro para el diagnóstico de artritis séptica?

A

Cultivo

439
Q

¿Cuál es la conducta a seguir en un diagnostico confirmado de artritis séptica?

A

Realizar drenaje del material purulento, el inicio de tratamiento antibiótico y dexametasona

440
Q

Tratamiento en pacientes sin riesgo para MRSA en artritis infecciosa

A

Dicloxacilina, cefotaxima o cefalotina + un aminoglucósido

441
Q

Tratamiento en pacientes con riesgo para SAMR en artritis infecciosa

A

Vancomicina + una cefalosporina de tercera generación

442
Q

Indicación de artrotomía evacuadora en artritis infecciosa

A

Lactantes y neonatos

443
Q

¿Cómo se define cistitis aguda y qué patógenos la causan?

A

infección del epitelio de la vejiga: disuria, frecuencia, urgencia, dolor suprapúbico, hematuria y orina maloliente. C trachomatis, N gonorrhoeae (uretritis), se acompañan de piuría, descarga vaginal o hematuria significativas.

444
Q

Diferencias entre pielonefritis aguda y crónica

A

Pielonefritis aguda → infección del aprenquima renal y el sistema colector: fiebre, escalofríos, dolor en flanco, diarrea, náusea y vómito añadidos a los síntomas de cistitis.

Pielonefritis crónica → inflamación, cicatrización y atrofia del parénquima renal causada por una infección persistente o recurrente; es más frecuente en px con reflujo vesicoureteral de evolución prolongada.

445
Q

Diferencias entre prostatitis aguda y crónica

A

Prostatitis aguda → infección de la próstata causada por el ascenso de una colonización del tracto urinario: fiebre de inicio abrupto, disuria, dolor pélvico o perineal, goteo y vacilación.

Prostatitis crónica → proceso más subágudo caracterizado por infecciones prostáticas recurrentes: ITU recurrentes, disuria o dolor pélvico.

446
Q

¿Cómo se define bacteriuria asintomática?

A

Presencia de un cultivo positivo en ausencia de síntomas. Su prevalencia general, en embarazo y es 3.5%, aumenta con la edad (7% a los 60a y 17% en >75a), embarazo y antecedente de ITU.
Dx → 2 cultivos en embarazo asintomáticas.

447
Q

Agentes causales de ITU

A
  1. E coli → 90% mujeres y 80% hombres
  2. Sthaphylococcus saprophyticus → mujeres sexualmente activas
  3. Klebsiella
  4. Enterococcus fecalis
448
Q

Estándar de oro para ITU y en qué pacientes se pide

A

urocultivo: px con dx poco claro, cistitis recurrente o recidivante, pielonefritis o ITU complicada.

449
Q

¿Cuantas UFC son necesarias en urocultivo para realizar diagnóstico de ITU en pediátricos?

A

Aspiración suprapúbica → 10 a la 3 CFU/ml
Cateterismo vesical → 10 a la 4 CFU/ml
Chorro intermedio:
- Gram (-) → 10 a la 5 CFU/ml
- Gram (+) → 10 a la 4 CFU/ml

450
Q

Síntomas más específicos de ITU

A

disuria, polaquiuria y urgencia miccional. La presencia de ≥2 de estos tiene VPP de 90% y se inicia tx empírico.

451
Q

Tratamiento de cistitis no complicada en la mujer

A
  1. TMP/SMX 160/800 mc c/12 hrs x 3d
  2. Nitrofurantoína 100 mg c/12 hrs x7 d
452
Q

Tratamiento de cistitis no complicada en pediátricos

A

Ambulatorio por 3-4 días

  1. TMP/SMX 8-12 mg de TPM/lg/d dividido en 2 dosis
  2. Nitrofurantoína 5-7 mg/kg/d dividido en 4 dosis
453
Q

Tratamiento de pielonefritis en pediátricos

A

cefalosporinas orales de 3ra, gentamicina (combinada con ampicilina si <3 meses).

454
Q

Tratamiento de cistitis y pielonefritis en hombres

A

TMP/SMX o flouroquinolonas x 10-14d.

455
Q

Tratamiento de pielonefritis en mujeres

A

(Previa toma de urocultivo): ambulatorio por 14d en px sin respuesta inflamatoria sistémica y que se espera que cumplan con el esquema →
1. Ciprofloxacino 500 mg c/12 hrs
2. TPM/SMX 160/800 mg c/12 hrs
Gram (+): amoxicilina/clavulanato o ampicilina

456
Q

Tratamiento de bacteriuria asintomática y cistitis en gestantes

A
  1. Fosfomicina 3 g DU
  2. Nitrofurantoína 100 mg c/6 hrs x 5 d (evitarlo en el 1er trimestre)
457
Q

Tratamiento de pielonefritis en gestantes

A

Ertapenem 1 g c/24 hrs x 10-14 d
Alternativo: ceftriaxona 2 gr c/24 hrs x 10-14 d

458
Q

¿Cómo se definen pielonefritis y cistitis enfisematosas?

A

Infecciones agudas necrosantes y caracterizadas por la formación de gas, que se localiza en la vejiga y su pared, en el caso de cistitis, y el tejido renal y circundante en la pielonefritis.

459
Q

Agentes causales en pielonefritis y cistitis enfisematosas

A

E coli 58% en cistitis y 45-52% pielonefritis
K pneumoniae 21%

460
Q

Cartacterísticas de Salmonella typhi

A

Bacilo gram (-) anaerobio facultativo, no especuladores, sensibles a la desecación, oxidasa negativos

461
Q

¿Cómo se adquiere salmonella?

A

Ingesta de alimentos contaminados → aves, huevos, lácteos
Más común en los meses más cálidos del año

462
Q

¿Qué síndromes produce la infección por salmonella?

A

Enterocolitis
Fiebre entérica
Bacteremia con infección metastásica
Estado portador cónico

463
Q

¿Cómo se diagnóstica fiebre tifoidea?

A

Considerarse en caso de fiebre >39° durante >72 hrs, cefalea, malestar general y tos no productiva; estreñimiento, diarrea, vómito, dolor abominal, exantema macular (rósela tifoídica), enantema lingual, hepatomegalia y esplenomegalia
Aislamiento → agar MacConkey, agar eosina-azul de metileno.

464
Q

¿Cómo se aborda la fiebre entérica?

A
  • Aglutinación de Widal: a partir de la 2da semana es positivo con un valor ≥1:160
  • Pruebas rápidas de detección de anticuerpos contra S. typhi a partir de la 2da semana
  • Hemocultivo → primera semana
  • Mielocultivo → si el hemocultivo es negativo
465
Q

Tratamiento de fiebre entérica

A

ciprofloxacino, cefixima, cloranfenicol, ampicilina, amoxicilina, TMP/SMX

En brote: azitromicina

466
Q

Agentes causales de NAC

A

El agente más frecuente es Streptococcus pneumoniae, seguido por Haemophilus influenzae y Staphylococcus aureus.
Px con EPOC es más común H influenzae y Moraxella catarrhalis.

467
Q

Clínica de NAC neumocócica

A

inicio súbito de escalofríos y fiebre con rigidez, tos productiva, dolor pleurítico e infiltrados focales.

468
Q

Estudio de primera linea en NAC

A

Rx tórax → las anormalidades son necesarias para establecer el dx, excepto en Pneumocystis jiroveci, donde una rx normal no lo descarta

469
Q

Tratamiento NAC no grave en niños

A

Ambulatorio.
- Amoxicilina VO
- Alergia a penicilina o sospecha de bacterias atípicas → macrólido (azitromicina) o doxiciclina

470
Q

Tratamiento NAG grave en niños

A

Hospitalizado.
Penicilina G benzatínica IV

471
Q

Prevención de neumonía

A

Vacunación → reduce 80% riesgo
Lavado de manos
Niños → lactancia materna y alimentos ricos en vitamina A

472
Q

Criterios de laboratorio para neumonía viral

A

Aislamiento del virus en cultivo celular
Detección directa del antígeno viral
Pruebas de amplificación del ácido nucleico

473
Q

Agentes más comunes de neumonía viral

A

Virus de influenza 8%
VSR 3%
PIV 2%
AV 2%

474
Q

Bacterias que causan meningitis

A

S pneumoniae, N mentngitidis, H influenzae, L monocytogenes, Salmonella, Nocardia, M tuberculosis, P aeruginosa, R rickettsii, Ehrlichia, T pallidum, Leptospira, B burdogferi

475
Q

Virus causantes de meningitis

A

enterovirus no polio, virus de la parotiditis, arbovirus, herpesvirus, virus de la coriomeningitis linfocítica, VIH, adenovirus, influenza, sarampión, síndromes posinfecciosos líricos.

476
Q

Hongos que causan meningitis

A

cryptococcus, histoplasma, coccidioides

477
Q

Nervio craneal más frecuentemente afectado en meningitis

A

VI - abducens

478
Q

Triada de meningitis

A

fiebre 77%, rigidez de nuca 83% y alteración del estado mental 69%

479
Q

Parámetros de LCR normal

A

Leucocitos 0-5: 0-15% neutrofilos
Glucosa 2/3 de la sérica: 45-81 mg/dl
Proteínas 15-50 mg/dl
Presión de apertura <180 cmH2O

480
Q

Tratamiento para meningitis

A
  • Ceftriaxona 2 gr IV c/12-24 hrs.
    >1 mes → 100 mg/kg/d dividido en 2 dosis
  • Cefotaxima 2 gr IV c/6-8 hrs
    1mes - 12 años o <50 kg → 200 mg/kg/d divido en 4 dosis
481
Q

Prodromo de encefalitis

A

Enfermedad vírica inesecífica que progresa a cefalea, fiebre y rigidez nucal, deterioro rápido y global en el estado mental; ataxia, hemiparesia, afasia, involucramiento de nervios craneales y psicosis. Convulsiones en el 50%.

482
Q

¿Por qué está contraindicada la PL en absceso cerebral y epidural?

A

Riesgo de herniación

483
Q

Agentes más frecuentes en EPI

A

C trachomatis (aguda) y N gonorrhoeae (subaguda)

484
Q

Síntomas característicos de EPI

A

Dolor en hipogastrio 90%
Descarga vaginal mucopurulenta 70%
Sangrado uterino anormal 40%

485
Q

Criterios diagnósticos de EPI

A

Evidencia histopatológica de endometritis en biopsia
USG que muestra inflamación en salpiques con o sin líquido libre en cavidad
Hallazgos laparoscopios de tumefacción y eritema de las salpinges → estándar de oro

486
Q

Estándar de oro para EPI

A

Hallazgos laparoscopios de tumefacción y eritema de las salpinges

487
Q

Escala Moni para severidad de EPI

A

Leve → no complicada, sin masa anexial, datos de abdomen agudo o irritación peritoneal
Moderada → complicada, presencia de masa anexial o absceso tubo-ovárico, con o sin signos de irritación peritoneal
Grave → diseminada a estructuras extrapélvicas, absceso tubo-ovárico roto o pelviperitonitis con manifestaciones de respuesta inflamatoria sistémica

488
Q

Tratamiento EPI en pacientes hospitalizadas

A

Ceftriaxona + doxiciclina +/- metronidazol x 14 días

489
Q

Tratamiento EPI ambulatorio

A

Ceftriaxona DU + doxiciclina +/- metronidaazol x 14 d

490
Q

¿Cómo se transmite el virus de la influenza?

A

gotas de flügge expulsadas al hablar, respirar o toser

491
Q

Grupos de alto riesgo en influenza

A

inmunocomprometidos, ancianos, y px con problemas respiratorios o cardiacos

492
Q

Manifestaciones de infección aguda de influenza en adultos

A

fiebre, malestar, mialgias, faringitis y tos no productiva por 7-10 días

493
Q

Periodo de incubación de virus influenza

A

1-4 días

494
Q

Clínica de infección gripal aguda por influenza en niños

A

fiebre elevada, síntomas intestinales (dolor abdominal y vómito), otitis media, miosotis y laringotraqueobronquitis.

495
Q

Prueba confirmatoria de influenza

A

PCR en tiempo real

496
Q

Tratamiento influenza AH1N1

A

Olsetamivir. inciar en las primeras 48 hrs

497
Q

Signos de alarma en niños con influenza

A

fiebre intensa, disnea, taquipnea, rechazo a la VO, convulsiones, alteración del estado de alerta

498
Q

Signos de alarma en adultos con influenza

A

disnea, vómito o diarrea persistente, trastorno del estado de alerta, deterioro agudo de la función cardiaca, agravamiento de una enfermedad crónica.

499
Q

¿En quienes se da quimioprofilaxis para virus de influenza?

A

Personal sanitario que haya estado en contacto cercano y sin equipo de protección con un caso probable, sospechoso o confirmado.
Personal de triage, urgencias, neurología, infectología, cuidados intensivos y con exposición a microgotas en todos los niveles de atención.
Individuos con alto riesgo de complicaciones que habiten lugares con riesgo alto de transmisión.

500
Q

Características microbiológicas de Vibrio Choleae

A

Bacilo curvo gram (-) anaerobio facultativo con un flagelo polar y diversos pili, oxidasa positivo

501
Q

¿Qué designa los serogrupos de V cholerae y cuales son?

A

LPS conformado por el lípido A (endotoxina) y una cadena de polisacárido O, que designa los serogrupos en:
Serogrupo O1: no produce cápsula, las infecciones que provoca no se extienden más allá del intestino:
Serotipos: Inaba, Ogawa e Hikojima
Biotipos:
Clásico → causante de la 6ta pandemia, produce infecciones sintomáticas y asintomáticas
El Tor → incluye las cepas de la 7ma pandemia (actual), infecciones asintomáticas.
Serogrupo O139: produce cápsula polisacárida → infecciones diseminadas

502
Q

Porcentaje de pérdida fetal por V Cholerae

A

50%

503
Q

¿Qué hace la toxina del cólera?

A

estimula la hiposecreción de electrolitos y agua, se une al receptor GM1m conformado por las subunidades A (interactúa con la proteína G aumentando AMPc) y B, codificadas por los genes ctxA y ctxB

504
Q

¿Cómo se realiza el cultivo en V Cholerae?

A

Debe realizarse al inicio de la enfermedad con muestras de heces frescas, con transporte Cary-Blair bajo refrigeración, en medios de agar sangre, agar MacConkey

505
Q

Tratamiento de V Cholerae

A

Reemplazo de líquidos, corrección de acidosis metabólica, reposición de pérdidas de L y reemplazo de pérdidas hídricas continuas.

506
Q

¿Qué se debe tener para dar de alta a un px con cólera?

A

Tolerancia oral: ≥1000 ml/hr
Gasto urinario satisfactorio: ≥40 ml/hr
Gasto fecal bajo: ≤400 ml/hr

507
Q

¿En qué agar crece Bordetella Pertusis?

A

Bordet-Gengou

508
Q

Enfermedad que causa Bordetella Pertusis

A

Tosferina

509
Q

¿Cómo se propaga la tos ferina?

A

Partículas aerosolizadas

510
Q

Estándar de oro para diagnóstico de tos ferina

A

Cultivo nasofaríngeo

511
Q

Fases de la tos ferina

A
  • Incubación: periodo asintomático de 7-10 días.
  • Catarral: 1-2 semanas, similar a un catarro común con rinorrea hialina, malestar general, fiebre, epífora, inyección conjuntiva, estornudos, anorexia y tos no productiva. Esta fase es la más contagiosa.
  • Paroxística: 2-4 semanas, se da la expulsión de las células ciliadas. Tos repetitiva y agotaste con estridor, vomito y leucocitosis.
  • Convalecencia: 3-4 semanas o más. Disminuye la tos, se desarrollan complicaciones secundarias:
    Neumonía 5-10%
    Convulsiones 1%
    Encefalopatía 0.1%
512
Q

Tratamiento de tos ferina

A

Medidas de soporte mantenimiento de la alimentación, oxigenación e hidratación adecuadas + azitromicina

513
Q

Cuáles son las indicaciones para lavarse las manos y reducir el riesgo de diarrea

A

Lavarse las manos después de aplicar, después del cambio de pañales, antes de preparar y comer alimentos

514
Q

Qué acciones de asaneamiento son las que se deben recomendar para reducir incidencia de diarrea

A

Lavado de manos eficaz, uso de desinfectante, a base de alcohol, en manos, te envío el agua para consumo humano, cocción de los alimentos, lavado de frutas, verduras, y de Numbers, disposición, disminución de basura y excretas

515
Q

Cuáles son los patógenos más importantes en diarrea aguda?

A

Rotavirus, criptosporidium, e coli, salmonela, shigella, Campilobacter j,  vivido cólera, giardia lambia, e histolytica

516
Q

Cuál es el patógeno que causa cólera

A

Vibrio cólera
Se produce una exotoxina en el intestino delgado y precipitación de agua y electrolitos

517
Q

Cuáles son los características clínicas de cólera

A

Diarrea, acuosa aguda como agua de arroz, vómitos, deshidratación, moderada, grave, muerte por choque si se trata en las primeras 24 horas

518
Q

La infección asintomática, por cólera, a qué serotipo se debe

A

Serotipo Thor, es la más frecuente, la forma moderada o grave se presenta en un 20%

519
Q

Quién se debe sospechar cólera, independientemente si la diarrea es agua de arroz

A

En pacientes de más de cinco años de edad con cinco o más evacuaciones diarreicas en 24 horas, cuyo cuadro clínico no sea mayor a cinco días de evolución(regla de los cinco)

520
Q

En quien utilizan los estudios diagnóstico en heces

A

Si están disponibles, en caso de disentería, deshidratación, moderada grave y síntomas que duran más de siete días

521
Q

En quien utilizan los estudios diagnóstico en heces

A

Si están disponibles, en caso de disentería, deshidratación, moderada grave y síntomas que duran más de siete días

522
Q

Cuál es la sintomatología específica de Yersinia

A

Pacientes con diarrea, enterocolítica, persistencia de dolor abdominal en niños, en edad escolar, dolor en cuadrante inferior, derecho, pacientes con fiebre y riesgo epidemiológico. 

523
Q

Pacientes con diarrea aguda, sin sangre en eses y deshidratación moderada. Cuál es la hidratación de elección

A

Hidratación viral, sin necesidad de antibióticos

524
Q

Pacientes con diarrea aguda, sin sangre en eses y deshidratación moderada. Cuál es la hidratación de elección

A

Hidratación viral, sin necesidad de antibióticos

525
Q

En qué paciente se recomienda el uso de antibióticos?

A

Pacientes en presencia de disentería

526
Q

Cuál es el primer paso del tratamiento de diarrea aguda?

A

Dilatación con solución de dilatación oral de osmolaridad reducida. Menor igual a 270mmol

527
Q

Cuáles son las indicaciones para refiere a segundo nivel a un paciente con deshidratación por diarrea

A

Deshidratación severa con sangre en meses, fiebre, dolor abdominal, duración de mayor a 48 horas, laboratorios confirmados

528
Q

Cuál es el tratamiento empírico para sospecha de diarrea del viajero

A

Ácido Med 1 g en una sola dosis o 500 mg una vez al día por tres días

529
Q

Un paciente con diarrea acuosa moderada grave, no asociada viajes y presencia de fiebre mayor a 38 cuál es la indicación?

A

Si la diarrea persiste por 72 horas, se hace cultivo y tratamiento empírico con azitromicina

530
Q

Diarrea crónica o persistente, cuáles son los principales patógenos más probables

A

Criptosporidium, Guardia, ciclospora,cystoispora, e histolytica

531
Q

Cuándo existe sangre visible meses, cuáles son los principales patogenos probables

A

Shigella, salmonela, Campilo, Bacter, e histolytica, vidrio no cólera, yersinia

532
Q

Cuándo existe dolor abdominal severo, eses sanguinolentas cuál es el patógeno probable

A

Ects, salmonela, shigella, Campilo Bacter, Yersinia,

533
Q

Cuando existe diarrea acuosa crónica con duración mayor o igual a un año. Cuál es el diagnóstico más probable

A

Se llama diarrea, Barinerd
 es un síndrome de intestino irritable pos infeccioso

534
Q

Cuáles son los antibióticos de elección en la enfermedad diarrea aguda

A

Levofloxacino 500 mg
Ciprofloxacino 500 mg o 750 mg
Ofloxacino 400mg
Azitromicina 1 g o 500 mg por tres días. Este se considera de elección empíricamente para de viajero.
Rifan piscina 200 mg

535
Q

Cuáles son los antibióticos de elección en la enfermedad diarrea aguda

A

Levofloxacino 500 mg
Ciprofloxacino 500 mg o 750 mg
Ofloxacino 400mg
Azitromicina 1 g o 500 mg por tres días. Este se considera de elección empíricamente para de viajero.
Rifan piscina 200 mg

536
Q

Cuál es el antibiótico de primera elección para cólera?

A
  1. Doxiciclina 300 mg. Dosis única
  2. Azitromicina 1 g. Dosis única.
  3. Ciprofloxacino 500 mg cada 12 horas por tres días o 2 g. Dosis única.
537
Q

Cuál es el antibiótico de primera elección para cólera?

A
  1. Doxiciclina 300 mg. Dosis única
  2. Azitromicina 1 g. Dosis única.
  3. Ciprofloxacino 500 mg cada 12 horas por tres días o 2 g. Dosis única.
538
Q

Cuál es el antibiótico de elección para shigelosis

A

Ciprofloxacino 500 mg cada 12 horas por tres días o 2 g en dosis única
Se selecciona de dos a 4 g. Dosis única.

539
Q

Cuál es el antibiótico de elección para shigelosis

A

Ciprofloxacino 500 mg cada 12 horas por tres días o 2 g en dosis única
Se selecciona de dos a 4 g. Dosis única.

540
Q

Cuál es el antibiótico de elección para amibiasis?

A

Metronidazol 750 mg

541
Q

Cuál es el antibiótico de elección para giardiasis

A

Metronidazol 750 mg, tres veces al día por cinco días

542
Q

Cuál es el antibiótico de elección para giardiasis

A

Metronidazol 750 mg, tres veces al día por cinco días

543
Q

Cuál es el antibiótico de elección para Campilo Bacter

A

Azitromicina 500 mg cada 24 horas por tres días 

544
Q

Cuál es la evaluación, según el método daca con el plan B de rehidratación

A

Paciente irritable o menos activo, ojos, hundidos, mucosa seca, sediento, turgencia de piel, reducida, pulso radial con bajo volumen
Debe tener por lo -2 signos, incluyendo pulso radial bajo y/o turgencia en la piel reducida